You are on page 1of 96

TABLE OF CONTENTS

Algebra of Propositions.... 5
P ropositions. 5
Exercises..... 8
Logical Operators.... ..... ........................................ 8
Exercises.. 10
Operation on Statements..... 10
Exercises.... 13
Log ical Identities... ..... .... ..... .... ..... ............................................... 13
Exercises.....

Chapter Test... .... .... ..... .............................................. 15

CHAPTER 2:QUANTlFlERS
Quantifiers

CHAPTER 1: ALGEBRA OF PROPOSITION

The Three Quantifiers...................... .


Exercises......... 21

Negation of Quantifiers..... 22
.
Exercises.. . 23
Chapter Test... 23

CHAPTER 3. BOOLEAN ALGEBRA


Boolean Algebra. 25

Exercises..... 26
2
The Basic Laws in Boolean Algebra..
Exercises..... . 28

Ten Basic Rules of Boolean Algebra.. 29


Exercises...... 29
De Morgan's Theorem..... 30
Exercises.... 31

Logic Gates... 31

Exercises..... 34

Chapter Test... 35

CHAPTER 4. LOGICAL REASONING


Logical Reasoning.. 37

Exercises..... 41
Analyzing Arguments using Truth Table. 4
Exercises..... . 43
Chapter Test....... 43

CHAPTER 5. SETS AND SUBSETS


Set. 45
Exercises..... 47
48
Set Theory
Exercises...... 50
Subset. 51
Solving Problem using the Concept of Set Theory including the Methods
of Venn 52
Diagram... ..... .... ..... .............................................................................................
Exercises..... . 5
Chapter Test....... 57

CHAPTER 6. BASIC SET OPERATIONS


Set Operation......................................................... 60
Exercises.. . 64
Algebra of Set.. 64
Exercises.....
Chapter Test.......

CHAPTER 7. SETS OF NUMBERS


Set.. 71
3
Basic Operation of Sets.. 72
Exercises..... . 78
Types of Sets...... 78
Exercises...... 83
Chapter Test....... 83

CHAPTER 8. FUNCTIONS
Function.. . 87
Exercises..... 89
Types of Function... 90
Exercises..... . 91
Chapter Test....... 92

CHAPTER 9. PRODUCT SETS & GRAPHS OF FUNCTIONS


Ordered Pair 94
Exercises..... . 95
Product Set... 95

Exercises..... . 96
The Graph of a Function...
Exercises..... 102
Chapter Test...... 103

CHAPTER 10. RELATIONS


Relation... ..... .... ..... .... ..... ..... 104

Exercises...... 105
Properties of Relation.... 105
Exercises..... 106
Relation represented by diagraph.. 106
Exercises..... . 106
Equivalence Relation.... 107
Exercises...... 108
The Well Ordering Principle. 108
Exercises..... 108
Transitive 109
Closure.... ..........................................................................................................
Exercises..... . 109
Chapter Test........................... 109

4
CHAPTER 1
Algebra of Proposítíons
Introduction:
In mathematics, we are in business of providing or disproving certain types of sentences.
As such, we are concerned with sentences that are either true or false. These are called
propositions.
This chapter presents the propositions, compound proposition, the use of logical
identities and the three primary logical operations such as AND, OR, and NOT.

Lecture Notes
What is Algebra of Proposition?
Algebra of Proposition is a branch of mathematical logic that deals with
propositions from the aspect of their logical meanings (true or false) and with logical
operations on them. The development of the algebra of logic was an attempt to solve
traditional logical problems by algebraic methods.

1.1. PROPOSITIONS
1.1.1. Proposition

In Logic, a proposition is a statement that affirms or denies something. The meaning


expressed in such a statement, as opposed to the way it is expressed. In Mathematics it is a
theorem to be demonstrated or a problem to be solved and a statement which has a truth value
either true (T) or false (F).
Examples of propositions are:

1. A bicycle has two wheels' that can be assigned a value of


2. A bicycle has ten wheels' that can be assigned a value of
I
3. The planet Earth is round' that can be assigned a value of T';
4. The planet Earth has the shape of a cube' that can be assigned a value of 'F.

1.12. Law of Dichotomy


An important property of propositions is that they satisfy the law of dichotomy, that is, a
proposition must have a logical value of either true (T) or false (F).
5
Thus, a statement that cannot clearly be assigned a logical value of either true (T) or false (F), is
not a proposition.

1.1B. Law of the excluded middle


Another important property of propositions is that they satisfy the law of the excluded
middle, that is a proposition cannot be simultaneously true (T) and false (F).

Thus, a statement that cannot unmistakably be assigned one, and only one logical value,
is not a proposition.

1.14. Proposition versus statements


It is important to stress at this point that not all statements are propositions.
An illustrative example of this, given by Dr Guillermo Miranda, is the statement
'It rains. '

Let us assume that it is raining today but that it will be sunny tomorrow. The statement
'It rains today' is then true (T), however the statement 'It rains everyday' is false (F). Thus, the
logical value (that is whether it is T or F) of the statement 'It rains' depends upon the context in
which it is said.

Therefore 'It rains' cannot be assigned clearly and unmistakably a unique logical value,
and thus it is not a proposition.

Thus, it is very important when one applies a mathematical property or proposition to


the development of models in natural sciences, to keep in mind the context or assumptions that
form part of the mathematical proposition, to ensure that it is applicable to the given model.

1.1.5. Equal propositions


Within propositional algebra, two propositions are said to be equal if and only if they
have the same logical value.

Thus, from this definition, given two propositions A and B, the following can be stated:
if A and B are both true (T) then they are equal, that is A = B; if A
and B are both false (F) then they are equal, that is A = B; if A is true (T)
B
and B is false (F) then they are not equal, that is ; if A is false (F) and B
is true (T) then they are not equal, that is A B

Note that the four items above cover all the possible combinations of the logical values of
propositions A and B; regardless of what propositions A and B actually are.

1.16. Examples
To illustrate the ideas, consider the propositions presented in section 1.11 and defining
propositions A, B, C, and D:

6
AE 'A bicycle has two wheels',
BE 'A bicycle has ten wheels',
CE 'The planet Earth is round',
DE 'The planet Earth has the shape of a cube';

it follows that that is 'A bicycle has two wheels'= The planet Earth is round',

since both A and C are true (T).

It also follows that that is 'A bicycle has ten wheels' 'The planet Earth has the shape of a cube',
since both B and D are false (F).
In turn, it then follows that that is 'A bicycle has two wheels'* 'The planet Earth has the
shape of a cube'
since A is true (T) and D is false (F).
It also follows that

that is A bicycle has ten wheels' The planet Earth is round'; since B is false (F) and
C is true (T).
Additionally, one can also state that that is 'A bicycle has two

wheels'* A bicycle has ten wheels';

since A is true (T) and B is false (F).

It also follows that that is The planet Earth is round'* 'The planet Earth has the shape of a cube'

since C is true (T) and D is false (F).

As a final example consider the following two propositions:

'A bicycle has eleven wheels' that can be assigned a value of


'A bicycle has twelve wheels' that can be assigned a value of 'F'
Since both propositions have the same logical value of false (F), it follows that both propositions
are equal in propositional algebra, that is

'A bicycle has eleven wheels'= 'A bicycle has twelve wheels'

EXERCISE:

A. Read the following set of statements. Identify whether it is true or false in its nature.
a. There are 12 months in a year.
b. There are 15 months in a year.
2. a. Rainbow has 6 colors.
b. Rainbow has 7 colors.
7
B. Using the four statements above, apply the concepts in equal proposition.

1.2. LOGICAL OPERATORS

What is a logical operator?


A logical operator defines a new proposition, S, from one or more given propositions A,
B,..., such that the logical value of the new proposition S depends on, and only on, the logical
values of the given propositions A, Be..
A logical operator is uniquely determined by its truth table. A truth table presents all the
possible combinations of logical values for the given propositions A, B,... and the corresponding
logical value of S for each combination.
12.1. The -1 operator (NOT)
Not A GA) is true when A is false, and vice-versa.

F T
Example using statement:

Statement: 6 is a prime number.


Not Operator. 6 is not a prime number; It is not true that 6 is prime.
1.22. The v operator (OR)
Given two propositions A and B, the new proposition A v B may be formed. Take note that A or B
is true if either A or B is true, including the case when both are true.

A B AAB
T T

Example using statement:

pvq
Let p=Washington is in Canada.
Let London is in England.
Or Operator: Washington is in Canada or London is in England.

12.3. The A operator (AND)


Given two propositions A and B, the new proposition A AB may be formed. Take note that, A
and B (AAB) is true when both A and B is true.
A B AAB
T T
F T
T T
8
Example using statement:
pnq
Let p=Washington is in Canada.
Let London is in England.
And Operator: Washington is in Canada and London is in England.

EXERCISE:
A. Draw a truth table for the following proposition: r
A-IS
2. -.u v v
3
B. Read the statements below. Using the 3 logical operators, give what is being asked.
1. Perform the -l operator in the following statement.
a. 5 is an even number.
b. 1 has no factor.
c. Earth is the fourth planet from the Sun.
2. Perform the v and A operator in the following statement.
a. Manila is the capital of the Philippines. Bangkok is the capital of
Thailand.
b. Mt. Fuji is in Japan.
Mt. Kilimanjaro is in Tanzania.
3. Using the following statements, perform what is needed. u- The sky is blue. v- The leaves
are green. w- The sun is yellow.
a.
b. -11_1 V v

c.

1.3. OPERATIONS ON STATEMENT


1.3.1 Negation
The negation of a statement A, denoted by ("not A"), is the statement that is
true if A is false, and false if A is true.
Example:

If p: "2 is even" then


1) "2 is not even".
2) If p: "2 5" then "2 + 2 5".
1.32 Conjunction
The conjunction of two statements A and B, denoted by A A B ("A and B"), is the
statement which is true if and only if A is true and B is true. We read it as "and".
9
1.3B Disconjunction

The disjunction of two statements A and B is a statement AvB which is true if and
only if A is true or B is true. Similarly, the disjunction of p and q has value false only
when both are false. We read it as "or".

1.34 Implication

The implication of p and q has value false only when p is true and q is false. It is
denoted by p q and read if p then q. A statement in the form p q is also called a
conditional statement, in which p is a sufficient condition for q and q is a necessary
condition for p.

1.3.5 Equivalence Statements

The equivalence statement p q is true only when p and q have the same value. It is read p
if and only if q and is also called a biconditional statement, in which p is a necessary and
sufficient condition for q, and vice versa.

Theorem: The following is a list of some common logical equivalence rules:

pvq = q vp

1.3.6. The Truth Table


A table showing what the resulting truth value of a complex statement is for all
the possible truth values for the simple statements. In using truth table, we will need to
use the operations on statements.
To review, the following symbols are shown below.

The Truth Table

Example:
10
Construct a

1.

2. Construct a truth table for the statement (mn-,p) —r


mn-,p

In this case, when m is true, p is false, and r is false, then the antecedent mn.p will
be true but the consequence false, resulting in an invalid implication; every other case
gives a valid implication.
Exercise:
A. Write the following propositions using x, y, and z.
Let x: "There are clouds. y: "It is raining. z: "It is not sunny. "
1. There are clouds if and only if it is raining.
2. If there are no clouds, then it is sunny.
3. Either it is raining or sunny.
4. If it is not sunny, then either there are clouds or it is raining.
5. It is not raining if and only if there are no clouds and it is sunny.
B. Construct a truth table for the statement:
vr

1.4. Logical Identities


Logical Identities

1. Identity Laws PAT p; pvF p


2. Domination Laws pvT ÞT; e
3. Idempotent Laws pvp p
11
qvp; ppxq
4. Double Negation Law
5. Commutative Laws pvq q,xp
6. Associative Laws (pvq)v r Pv (qvr); (pnq) A r p A (qnr)
7. Distribution Laws

8. De Morgan's Laws

9. Miscellaneous or Tautology
10. And Contradiction
11. Implication Equivalence
12. Bicondtional Equivalence
The Proof Process

Logical Steps
-Definitions
-Already-proved equivalences
-Statements (e.g., arithmetic
or algebraic)

Example:

1. Prove: (PA-,q) v q pvq


Solution:

Left-Hand Statement
Commutative
v—-q) Distributive
Or Tautology
Identity
pvq Commutative

12
Take note: Begin with exactly the left-hand side statement. End with exactly what is on the right.
Justify EVERY step with a logical equivalence

EXERCISE:
prove:

CHAPTER TEST:
A. Write the following propositions using p, q, r.
Let p: "Today is cold", q: "Today is hot", and r: "Today is windy".

a) Today is hot if and only if not windy.


b) Either today is cold or not cold.
c) If today is not windy then it is not hot.
d) Today is neither cold nor windy.
e) If today is windy then either it is hot or cold.

B. Draw the truth table for each of the following propositions.

C. Draw the truth table for each of the following propositions USING (<

1. p: (5 is a positive integer) q; (4 > 5)

2. p: (3 < 4); q: (9-2 6)


3. p: (4 4); q: (9—2 = 6)

4. p: (4 > 5); q.(9-2 = 6)

5. p: (2 + 2 4); q: (2 = 4-2)
6.
D. Write an equivalent statement using contrapositive.
a) If I study hard then I get good mark.
b) If it rains then it is not hot.

13
c) If today is not Sunday then tomorrow is not Monday.
d) If I am not lazy then I come to the lecture.
E. Prove by applying the logical equivalence rules.

F. True or False. Prove by any method you like.

G. Using logical identities,


1. Prove: p p v q is a tautology.
2. Prove: (pnq) p is a tautology.

3. Prove or Disprove: p q pA ???

Reference:

https://iopscience.iop.org/book/978-1-6432-7016-6/chapter/bk978-1-6432-7016-6ch1

https://courses.lumenlearning.com/math41ibarts/chapter/truth-tables-and-
analyzingarguments-examples/
CHAPTER 2
Quantífíers
Introduction:
Remember that the sentence "x is an American," is not a proposition. This is because the
truth of the statement changes based on different values for x.x. Such a sentence is called an
open sentence or predicate However, it is quite useful to include variables in our statements, so
our logic should be able to accommodate that. To this end we introduce quantifiers quantifier
is a symbol which states how many instances of the variable satisfy the sentence.
This chapter presents the logical connectives, predicates and quantifiers.

14
Lecture Notes
What is Quantifier?

A quantifier is a word or phrase which is used before a noun to indicate the amount or
quantity: 'Some', 'many', 'a lot of' and 'a few' are examples of quantifiers.
Quantifiers can be used with both countable and uncountable nouns. There is a large
quantity of fish in this river. He's got more friends than his sister.

2.1 The Three Quantifiers


21.1. Universal Quantifier

The universal quantifier, V, reads "for all". If E is a set and P is a predicate defined on
elements of E (that is, a Boolean-valued function P: E {true, false}), then the assertion

states that P(x) is true for all elements x of E.

Example

l. vx(x 2 0), i.e., "the square of any number is not negative."


2. VxVy(x + y = y + x), i.e., the commutative law of addition.
3. + y) + z = x(y + z)), i.e., the associative law of addition.
The "all" form. The universal quantifier is frequently encountered in the following context:

which may be read, "All x satisfying P(x) also satisfying Q(x)". Parentheses are crucial here; be
sure you understand the difference between the "all" form and

which may be read, "All x satisfying P(x) also satisfy Q(x)." Parentheses are crucial here; be sure
you understand the difference between the "all" Q(x)), and
Q(x).

The latter formula might also be written as Q(x))which is to say that the
universal quantifier has higher precedence than the conditional; to avoid misunderstanding, it is
best to include the parentheses. The meaning of this formula might not be clear at first. The x in
15
P(x) is bound by the universal quantifier, but the x in Q(x) is not. The formula
Q(x))has the same meaning as (Vx(P(x) Q(y) and its truth depends on the value assigned to the
variable in Q(•).

Example:

1. Vx(x is a square x is a rectangle), i.e., "all squares are rectangles.


2. Vx(x lives in Walla Walla x lives in Washington), i.e.," every person who lives in Walla
Walla lives in Washington."

This construction sometimes is used to express a mathematical sentence of the form "if this,
then that," with an "understood" quantifier.

Example:

1. If we say, "if x is negative, so is its cube," we usually mean "every negative x has a
negative cube". This should be written symbolically as < 0) (x 3 < 0) )
2. "If two numbers have the same square, then they have the same absolute value" should
be written as = y2) (IXI = IYI
If S is a set, the sentence "every x in S satisfies P(x)" is written formally as

For clarity and brevity, this is usually written Vx e S(P(x). To understand and manipulate the
formula Vx e S(P(x) properly, you will sometimes need to "unabbreviate" it, rewriting it as
e s P(x))
Example:

1. Vx e x) stands for Vx(x e [0,1] x)


1. Vx < O(lxl = —x) stands for Vx(x < 0 IXI = —x)

2.12. Existential Quantifier


The existential quantifier, 3, reads "it exists". If E is a set and P is a predicate defined
on the elements of E, then the assertion

states that there exists an element x of E such that P(x) is true.

Example. Let P(n) be a predicate defined on the set of integers by "n is even". The
assertion an, P(n) states that there exists an even integer. The assertion which is false,
states that all integers are even.

We will sometimes use a modified version of 3 which states existence of a unique


element of a set satisfying a given predicate.

Another example:

16
• 3x(x x 2 ) is true since x = 0 is a solution. There are many others.
• ax By (x 2 + Y 2 = 2xy) is true since x = y = 1 is one of many solutions.

The "some" form. The existential quantifier is frequently encountered in the following
context:
3x(P(x) A Q(x)), which may
be read, "Some x satisfying P(x) also satisfies Q(x).
Example.
• 3x(x is a professor Ax is a republican), i.e., "some professor is a republican.'
• 3x(x is a prime number A x is even), i.e., "some prime number is even."

It may at first seem that "Some x satisfying P(x) satisfies Q(x)" should be translated as

3x(P(x) Q(x))
like the universal quantifier. To see why this does not work, suppose P(x) = "x is an apple" and
Q(x) = "x is an orange". The sentence "some apples are oranges" is certainly false, but

3x(P(x) Q(x))
is true. To see this suppose xo is some particular orange. Then P(xo) Q(xo) evaluates to FST,
which is T, and the existential quantifier is satisfied.

We use abbreviations of the "some" form much like those for the "all" form.

Example:

• ax < 0(x 2 = 1) stands for 3x((x < 0) A (x 2 = 1)


• ax e [O,1](2x 2 + x = 1) stands for 3x((x e [0,1]) A (2x 2 + x = 1))
If V corresponds to "all" and 3 corresponds to "some" do we need a third quantifier to
correspond to "none"? As the following shows, this is not necessary:

Example:

• "No democrats are republicans", can be written as Vx(x is a democrat x is not a


republican).
• "No triangle are rectangles," can be written Vx(x is a triangle x is not a
rectangle).

In general, the statement "no x satisfying P(x) satisfies Q(x)" can be written

17
mQ(x))
(You may wonder why we do not use A Q(x)). In fact, we could—it is equivalent to 3x(P(x)
Q(x)),
21.3. The Existential Quantifier (followed by the exclamation point)
The existential quantifier followed by the exclamation point, 3!, reads "there is a
unique". If E is a set and P a predicate defined on elements of E, then the assertion
31. X E E, P (X)

states that there exists a unique element x of E such that P(x) is true.

Example: Let P(x): x + 2 = 5.

a. Vx P(x): "for all real numbers x, x + 2 = 5", which is false.


b. ax P(x): "there is a real number x such that x + 2 = 5", which is true.
c. 3! x P(x): "there is a unique real number x such that x + 2 = 5", which is true. d ax
P(x): "there is a real number x such that x + 2 = 5".
-Ax P(x): "there is no real number x such that x + 2 = 5" which is equivalent to Vy -
IP(x): "for all real numbers x, x + 2 5".
g. Vx P(x): "for all real numbers x, x + 2 = 5".
h. --,Vx P(x): "not all real numbers x satisfies x + 2 = 5" which is equivalent to By
"there is a real number x such that x + 2 5".

EXERCISE:

In these problems, assume the universe of discourse is the real numbers.

A. Express the following as formulas involving quantifiers:

a) Any number raised to the fourth power is non-negative.

b) The sine of an angle is always between +1+1 and —1-1.

c) The secant of an angle is never strictly between +1+1 and —1-1.

B. Suppose X and Y are sets. Express the following as formulas involving quantifiers.

a) Every element of X is an element of Y.

b) Some element of X is an element of Y.

d) No element of X is an element of Y.
C. Express the following laws symbolically:

a) the commutative law of multiplication

b) the associative law of multiplication


18
c) the distributive law

D. Are the following sentences true or false?

1. Vx Vy(x < y x2 < y 2)

2. ax < 03y < O(x 2 + xy + y 2

E. Suppose P(x) and Q(y) are formulas.

1. Is Vx Vy(P(x) Q(y)) equivalent to Vx(P(x) VyQ(y))?

2.2. Negation of Quantifiers.


Negation of quantifiers. If P(x) is a predicate, then ax, P(x) and Vx, P(x) are statements. We can
therefore combine these statements using the logical operations v, A and -l to form more
complex statements. It should be noted that the following rules should be applied when
determining the negation of a quantified statement.
Proposition. Let E be a set, and P a predicate defined on the elements of E. Then

Thus if E and F are two sets and P a predicate of two variables x e E and y e F, the negation Of vx
E E,3y E is ax E X,Vy E F, —P (x,y) .

Therefore, the negation of


All sheeps are black
is the statement:

There exists a sheep that is not black


and reversely.
Similarly, the negation of
All men have at least one friend
is
There is a man who has no friends.
Indeed, if E is the set of men and P(x,y) denotes "y is a friend of x" then the first statement
translates into

E E,3y E ,
and its negation is therefore

19
EXERCISE:
Negate the following statements then denote it using symbols.

1. All dogs are poodles.


2. Some books have hard covers.
3. No U.S. presidents were residents of Georgia.
4. Some cats are mammals.
5. Some cats aren't mammals.

CHAPTER TEST.
A. Let P(x): x < 2x.
a) What is the value of Vx P(x)?
b) What is the value of ax P(x)?
c) What is the value of 3! x P(x)?

B. Let P (x, y): + = (x + Find the values of the following propositions.

c) Vx ay P (x, y)
d) By Vx P (x, y)
e) Vy ax P (x, y)
C. Repeat the Problem in B employing ! instead of 3
D. Repeat the same problem in B using the following predicates.
a) P (x, y): x2 + y2 > 0

E. Suppose P(x) and Q(y) are formulas.

a) Is ax 3y(P(x) A Q(y)) equivalent to ax (P(x)) A 3yQ(y))?

Reference:

https://www.whitman.edu/mathematics/higher math online/section01.02.html

https://www.math.fsu.edu/œwooland/hm2ed/Part2Module1/Part2Module1.pdf

20
CHAPTER 3
Boonean AÕebra
Introduction:
George Boole, a nineteenth-century English Mathematician, developed a system of logical
algebra by which reasoning can be expressed mathematically. In 1854, Boole published a classic
book, "An Investigation of the Laws of thought" on which he founded the Mathematical theories
of Logic and Probabilities.
This chapter presents the Boolean Algebra and its rule, the basic laws of Boolean algebra.
It will also discuss the ten basic rules of Boolean algebra, the De Morgan's theorem, and the
Logic gates.

Lecture Notes
WHAT IS A BOOLEAN ALGEBRA?
3.1. BOOLEAN ALGEBRA
Boolean logic is an abstract mathematical structure. Boolean Algebra is the calculus of
logic. It is used to analyze and simplify the digital (logic) circuits. It uses only the binary numbers
i.e. 0 and 1. It is also called as Binary Algebra or logical Algebra. Boolean algebra was invented by
George Boole in 1854. Boole tried to formalize the process of logical reasoning using symbols
instead of words. Boolean Algebra provides us a basic logic for the operations on binary
numbers 0, 1. Therefore, Boolean Algebra an essential tool since telephone, computers and
many kinds of electronic control devices are based on a binary system, this branch of
Mathematics are very useful for the internal working.
Boolean refers to a system of logical thought that is used to create true/false statements.
1 is considered to be true because it is non-zero. The fourth expression assigns a value of 0 to 1.
0 is considered to be false.

3.1.1. Rule in Boolean Algebra


Following are the important rules used in Boolean algebra.
21
Variable used can have only two values. Binary 1 for HIGH and Binary 0 for LOW.
Complement of a variable is represented by an overbar G). Thus, complement of variable
B is represented as B . Thus if B = 0 then B = 1 and B = 1 then B = 0.
ORing of the variables is represented by a plus (+) sign between them. For example,
ORing of A, B, C is represented as A + B + C.
Logical ANDing of the two or more variable is represented by writing a dot between
them such as A.B.C. Sometime the dot may be omitted like ABC.

EXERCISE:
Boolean algebra is a strange sort of math. For example, the complete set of rules for
Boolean addition is as follows:

0+1=1
1+0=1
1+1=1
Suppose a student saw this for the very first time, and was quite puzzled by it.
What would you say to him or her as an explanation for this? How would in the world
can 1+1=1 and not 2? And why are there no more rules for Boolean addition? Where is
the rule for 1+2 or 2+2?

2. The following set of mathematical expressions is the complete set of times tables for the
Boolean number system:

oxl=o
1 xo-o
1 xl=l
Explain how this can be so, being that there is no statement saying 1 = 2 or 2 x-3 = 6 Where
are all the other numbers besides 0 and 1?

3.2. THE BASIC LAWS OF BOOLEAN ALGEBRA


32.1. Commutative Law

allowing a change in position for addition and multiplication. Any binary operation
which satisfies the following expression is referred to as commutative operation.
Examples:

A-B=B-A

22
Commutative law states that changing the sequence of the variables does not
have any effect on the output of a logic circuit.

3.22. Associative Law

allowing the removal of brackets for addition and multiplication. This law states
that the order in which the logic operations are performed is irrelevant as their effect is
the same.

Examples:

i. A-(B-C) ii. (A+B)+


C- A+(B+C)
32.3. Distributive Law

allowing the factoring of an expression, are the same as in ordinary algebra.


Distributive law states the following condition.

Examples:

Here is list of properties of Boolean Algebra


• associativity of A: AA(BAC) = (AAB) AC.

• associativity of v: Av(BvC) = (AvB) vC.

• commutativity of A: AAB = BAA.

• commutativity of v: AvB = BvA.

• distributivity of A with respect to v: AA(BvC) = (AAB) v(AAC).

• the true statement 1 is a neutral element for the operation A, and the false statement 0 is a
neutral element for v: for any statement A we always have

• 0 annuls all statements: for any assertion A we have

AMO O.

EXERCISE:
A. Determine what kind of Boolean law is being applied.

23
A-(B-C)

3. AvB = BvA

4. AA(BAC) = (AAB)

5. A+B=B+A
B. The Law of Distribution in Boolean algebra is identical to the law of distribution in
"normal" algebra:

A(B+ C) AB+AC Applying the Law of Distribution

While the process of distribution is not difficult to understand, the reverse of distribution
(called factoring) seems to be a more difficult process for many students to master: AB+AC=
A(B+ C) Factoring A out of each term
Survey the following examples of factoring, and then describe what this process entails.
What pattern(s) are you looking for when trying to factor a Boolean expression?

CD + AD + BD D(C+ A + B)

XY Z + XYZ = + XO

AB + ABCD + BCD + B + ACD + CD + 1)

3.3. Ten Basic Rules of Boolean Algebra

Anything ANDed with a 0 is equal to 0. A * 0 = 0


2. Anything ANDed with a 1 is equal to itself. A * 1 = A
3. Anything ORed with a 0 is equal to itself. A + 0 = A
4. Anything ORed with a 1 is equal to 1. A + 1 = 1
5. Anything ANDed with itself is equal to itself. A * A = A
6. Anything ORed with itself is equal to itself. A + A = A
7. Anything ANDed with its own complement equals 0. A *
8. Anything ORed with its own complement equals 1. A ± A— 1 9. Anything complemented-
twice is equal to the original. A = A
10. The two variable rule. A+AB=A+B

EXERCISE
A.Determine which rule is being used in each step of the following Boolean simplification:

24
AB+BB+BC+BC
AB BC

AB+B+C
B A student makes a mistake somewhere in the process of simplifying the following Boolean
expression:

AB+A(B+C) AB+AB+C
AB+C

Determine where the mistake was made, and what the proper sequence of steps should be to
simplify the original expression.

3.4. De Morgan's Theorem

De Morgan's theorem allows large bars in a Boolean Expression to be broken up into


smaller bars over individual variables.
De Morgan's theorem says that a large bar over several variables can be broken between
the variables if the sign between the variables is changed.
De Morgan's theorem can be used to prove that a NAND gate is equal to an OR gate with
inverted inputs. De Morgan's theorem can be used to prove that a NOR gate is equal to
an AND gate with inverted inputs.
In order to reduce expressions with large bars, the bars must first be broken up. This
means that in some cases, the first step in reducing an expression is to use De Morgan's
theorem.
It is highly recommended to place parentheses around terms where lines have been
broken.

AB=A+B

A+B=AB

For example:

Apply De Morgan Theorem to remove the overbar covering both terms from the expression.

X=C+D
To apply DeMorgan's theorem to the expression, you can break the overbar covering both
terms and change the sign between the terms. This results in
Deleting the double bar gives

25
EXERCISE:

Use DeMorgan's Theorem, as well as any other applicable rules of Boolean algebra, to simplify
the following expression so there are no more complementation bars extending over multiple
variables:

AB +

2. J + KJL

3. XYZY

3.5. Logic Gates


Logic gates are the basic building blocks of any digital system. It is an electronic circuit
having one or more than one input and only one output. The relationship between the input
and the output is based on a certain logic. Based on this, logic gates are named as AND gate, OR
gate, NOT gate etc.
3.5.1. AND Gate

The AND gate receives two inputs a and b, and produces an output denoted by anb. The
truth table for an AND gate is as follows:

Input Output

a b
o o AND
gere

o
1
The only way that the output can be 1 is when a AND b is both 1. In other word there are needs
to be an electrical pulse at a AND b before the AND gate will output an electrical pulse.

3.52. OR Gate
The OR gate receives two inputs a and b, and produces an output denoted by AvB. The
truth table for an OR gate is as follows:

26
Input
Output
a b avb

o o o

This is an OR gate

The output will be one when an a and b or both are 1.


3.5.3. NOT Gate

NOT gate is also known as Inverter A not gate is capable of reversing the input pulse. The
truth table for a NOT gate is as follows:

Input a —a
a
This is a NOT
gate

The NOT Gate receives an input either a pulse (1) or no pulse (0) and produces an output
that follows:

If input a is 1, the output is 0.


If input a is 0, the output is 1.
These three gates, NOT, AND, and OR, can be joined together to form combinatorial
circuits to represent Boolean expression.

Example:

Use logic gates to represent

a.

Solution:

Draw up the truth table for each circuit.

27
b. (xvy)

Solution:

Draw up the truth table for each circuit.

Y xvy (xvy) „Avx

EXERCISE:

A. Write the Boolean expression for each of these logic gates, showing how the output (Q)
algebraically relates to the inputs (A and B):

B Convert the following logic gate circuit into a Boolean expression, writing Boolean sub-
expressions next to each gate output in the diagram:

28
Chapter Test:
Identify each of these logic gates by name, and complete their respective truth tables:

OutB1t Output Output

Outvot A B A B OutF0t
o o

Ou tput Output
Output

Outvot A Output

Output Output
O A B Out A B Outvot
ut put
p
at

Use logic gates to represent these expressions and draw up the corresponding truth table.

2. ave bnc)

Write down the Boolean expression for each of the circuits below:

4.

29
5.

Give the relationship that represents the dual of the Boolean property A + 1 = P (Note: *
= AND, + = OR and NOT)

5.

Reference:
https://www.tutorialspoint.com/computer logical organization/boolean algebra.htm
https://pbte.edu.pk/text% 20books/dae/math 123/Chapter I I .pdf
http://www.cimt.org.uk/projects/mepres/alevel/discrete chl l.pdf
https://www.allaboutcircuits.com/worksheets/boolean-algebra/ http
://www.ee.surrey.ac.uk/Projects/Labview/boolalgebra/q uiz/index.html
https://grace.bluegrass.kctcs.edu/—kdunnOOOl/files/Simplification/4 Simplification
print.html https://www.siue.edu/—iloreau/courses/math-223/notes/sec-quantifiers.html
https://www.allaboutcircuits.com/worksheets/boolean-algebra/

CHAPTER 4
£ogícafReasoníng
Introduction:
Logic may be defined as the science of reasoning. However, this is not to suggest that
logic is an empirical (i.e., experimental and observational) science like physics, biology or
psychology. Rather, logic is a non-empirical science like mathematics. Also, in saying that, logic is
the science of reasoning, we do not mean that is concerned with the actual mental (or physical)
process employed by thinking entity when it is reasoning. The investigation of the actual
reasoning process falls more appropriately within the province of psychology, neurophysiology,
or cybernetics. Distinguishing correct reasoning from incorrect reasoning is the task of logic.

30
This chapter presents the argument and its components, constructing truth tables,
proving and disproving hypothesis, evaluating the truth of statement and the different types of
reasoning.

1. Isolate and identify the various components of any given argument.


Learning 2. Construct truth tables, prove or disprove a hypothesis, and evaluate the
Objectives truth of a statement using the principles of logic.
3. Discuss the different types of reasoning.

Lecture Notes
4.1 Logical Reasoning?
Logical reasoning is a form of thinking in which premises and relations between premises
are used in a rigorous manner to infer conclusions that are entailed (or implied) by the premises
and the relations. It is the umbrella term for at least three different types of reasoning. These
are known as deductive reasoning, inductive reasoning and abductive reasoning and are based
on deduction, induction and abduction respectively.
41.1. What is an Argument?
Logical argument is a claim that a set of premises support a conclusion.
It is also a collection of statements, one of which is designated as the conclusion,
and the remainder of which are designed as the premises.

Note that it is not a definition of a good argument. Also, note that in the context of
ordinary discourse, an argument has an additional trait.

Usually, the premises of an argument are intended to support orjustify the


conclusion of the argument.

The building blocks of a logical argument are propositions, also called statements. A proposition
is a statement which is either true or false. For example,
"Washington D.C. is the capital of the United States."
An argument is connected series of statements that create a logical, clear, and defined
statement. There are three stages to create a logical argument: Premise, inference, and
conclusion.

Stage 1: Premise
The premise defines the evidence, or the reasons, that exist for proving your statement.
Premises often start with words like "because", "since", "obviously" and so on.
31
Stage 2: Inference
In this stage, we take one or more statements that are agreed upon as true and come up
with a new statement. This new statement often uses the words "implies", "suggest" or
"therefore".

Stage 3: Conclusion
The conclusion of an argument is a final statement. The conclusion is often stated as the
final stage of inference. It's validity, or truth, is confirmed by the truth of the premises
statement(s) and inference statement(s). Conclusions use words like "therefore', "it follows
that" and "in conclusion".
A Logical Arguments Example:

• Premise Statements:
"Washington D.C. is the capital of the United States."
"The President of the United States lives in the White House."
"The White House is at 1600 Pennsylvania Avenue, Washington D.C."

• Inference Statements:

"These statements imply that the White House is located in Washington D.C."
"These statement also imply that the President lives in Washington D.C. by living in the White
House."

• Conclusion Statement:

Therefore, the President of the United States lives in the capital of the United States.
4.12. Two general types of arguments: inductive and deductive arguments.
An inductive argument uses a collection of specific examples as its premises and uses them to
propose a general conclusion.
Ex: "The last mayor was honest. The current mayor is honest. All mayors are honest."

A deductive argument uses a collection of general statements as its premises and uses them to
propose a specific situation as the conclusion.
Ex: "Every word has the letter e in it. Therefore, your name has the letter e in it."

For instance,
The argument "when I went to the store last week I forgot my purse, and when I went
today I forgot my purse. I always forget my purse when I go the store" is an inductive
argument.
The premises are:

32
I forgot my purse last week I
forgot my purse today The
conclusion is:
I always forget my purse
Notice that the premises are specific situations, while the conclusion is a general statement. In
this case, this is a fairly weak argument, since it is based on only two instances.
The argument "every day for the past year, a plane flies over my house at 2pm. A plane
will fly over my house every day at 2pm" is a stronger inductive argument, since it is
based on a larger set of evidence.

41.3. EVALUATING INDUCTIVE AND DEDUCTIVE ARGUMENTS


An inductive argument is never able to prove the conclusion true, but it can provide
either weak or strong evidence to suggest it may be true.
A deductive argument is considered valid if all the premises are true, and the conclusion
follows logically from those premises. In other words, the premises are true, and the
conclusion follows necessarily from those premises.

EXAMPLE
The argument "All cats are mammals and a tiger is a cat, so a tiger is a mammal" is a valid
deductive argument.

The premises are:


All cats are mammals
A tiger is a cat

The conclusion is:


A tiger is a mammal
Both the premises are true. To see that the premises must logically lead to the conclusion, one
approach would be use a Venn diagram. From the first premise, we can conclude that the set of
cats is a subset of the set of mammals. From the second premise, we are told that a tiger lies
within the set of cats.
From that, we can see in the Venn diagram that the tiger also lies inside the set of mammals, so
the conclusion is valid.

33
2. Premise: All firefighters know CPR Premise:
Jill knows CPR
Conclusion: Jill is a firefighter
From the first premise, we know that firefighters all lie inside the set of those who know
CPR. From the second premise, we know that Jill is a member of that larger set, but we do not
have enough information to know if she also is a member of the smaller subset that is

firefighters.

Since the conclusion does not necessarily follow from the premises, this is an invalid argument,
regardless of whether Jill actually is a firefighter.
*It is important to note that whether or not Jill is actually a firefighter is not important in
evaluating the validity of the argument; we are only concerned with whether the premises are
enough to prove the conclusion.

EXERCISE
Using the three stages (premises, inference and conclusion), create your own logical
argument. (At least two arguments)
2. Determine if the following statements is deductive or inductive.
a. It's sunny in Singapore. If it's sunny in Singapore, then he won't be carrying an
umbrella. So, he won't be carrying an umbrella.
b. Every time I've walked by that dog, it hasn't tried to bite me. So, the next time I walk
by that dog it won't try to bite me.

34
3. Evaluate the following deductive/inductive argument. Include a diagram for your answer.
a. Premise 1: All dogs are mammals.
Premise 2: All collies are mammals.
Conclusion: All collies are dogs
b. Premise 1: All cats are mammals.
Premise 2: All mammals are animals. Conclusion:
All cats are animals.

4.2. ANALYZING ARGUMENTS USING TRUTH TABLES

To analyze an argument with a truth table:


1. Represent each of the premises symbolically
2. Create a conditional statement, joining all the premises with and to form the antecedent,
and using the conclusion as the consequent.
3. Create a truth table for that statement. If it is always true, then the argument is valid.

Example:
Consider the argument:
1. Premise: If you bought bread, then you went to the store Premise:
You bought bread
Conclusion: You went to the store
While this example is hopefully fairly obviously a valid argument, we can analyze it using a truth
table by representing each of the premises symbolically. We can then look at the implication
that the premises together imply the conclusion. If the truth table is a tautology (always true),
then the argument is valid.
We'll get B represent "you bought bread" and S represent "you went to the store". Then the
argument becomes:

Premise: B SB—S
Premise:

Conclusion: s
To test the validity, we look at whether the combination of both premises implies the
conclusion; is it true that

35
Since the truth table for is always true, this is a valid argument.

EXERCISE:
Consider the following arguments. Analyze it and prove using truth tables. The variables to be
used were indicated in each number.
1. Premise: If I go to the mall, then I'll buy new jeans
Premise. If I buy new jeans, I'll buy a shirt to go with
it
Conclusion: If I got to the mall, I'll buy a shirt.
Let M = I go to the mall, J = I buy jeans, and S = I buy a shirt.
2. Premise: If I go to school, then I'll
Premise: graduate.
Conclusion: If I graduate, I'll have a job.
If I go to school, I'll have a job.
I graduate and J- I have a job.

CHAPTER TEST.
A. Differentiate inductive and deductive argument.
B. Show the stages of creating logical argument using a graphic organizer. Give example in
each stage.
C. Consider the arguments. Identify and prove if it is valid or invalid argument.
1. Premise: If you live in Seattle, you live in Washington.
Premise: Marcus does not live in Seattle.
Conclusion: Marcus does not live in Washington.
2. "You are a married man, so you must have a wife. D. Using truth table,
analyze the arguments.
Premise: If I work hard, I'll get a raise.
Premise: If I get a raise, I'll buy a boat.
Conclusion: If I don't buy a boat, I must not have worked hard.
E. Premise 1: Most peacocks eat oatmeal for breakfast.
Premise 2: This bird is a peacock.
Conclusion: Therefore, probably this bird eats oatmeal for breakfast.

References:
36
https://www.fibonicci.com/logical-reasoning/
https://www.lead4change.org/2014/03/06/taking-lesson-crafting-logical-argument/
https://link.springer.com/referenceworkentry/lO.1007%2F978-l-4419-1428-6 790
https://courses.lumenlearning.com/wmopen-mathforliberalans/chapter/building-arguments-
withlogic/ https://iep.utm.edu/ded-ind/
https://courses.Iumenlearning.com/atd-pima-philosophy/chapter/I -2-arguments-types-
ofreasoning/
https://courses.lumenlearning.com/math41ibarts/chapter/truth-tables-and-analyzing-
argumentsexamples/

37
CHAPTER 5
Sets aníSubsets
Introduction:
The notion of set in mathematics is ancient. However, the modern theory of sets was
born in the 1870s under the impetus of mathematicians like Georg Cantor (1845-1918) and
Richard Dedekind (1831-1916). The discoveries of paradoxes such as Russell's one led in the
beginning of the 20th century to a formalization of the theory of sets and a discovery of
different axiomatic formulations, including that of Zermelo-Frankel which is the most widely
used nowadays. The introduction by Cantor of the Theory of Cardinals for infinite sets has been
considered as a genuine revolution in the late nineteenth century.
This chapter presents the concept of sets and subsets. It will also deal in solving a
problem which include the use of Venn diagrams. This chapter includes the operations and
properties on sets and subsets as well.

Lecture Notes
5.1 Set

Set is an organized collection of objects, things or symbols with common characteristics that
are clearly defined. The individual objects in a set are called the members or elements of the
set. There are three ways to describe sets:

Listing the Elements (Rooster Method)

The set can be defined by listing all its element, separated by commas, and enclosed
within braces.
Examples:

P = {v, w,x,y,z}
2. Describing The Elements
The set can be defined, where possible, by describing the elements clearly in
words.

38
Examples:

E is the set of the multiples of 5.


M is the set of vowels in the English alphabet.
Z is the set of months of a year.
3. Description by Set Builder Notation
The set can be defined by describing the elements using mathematical
statements. This is called the set-builder notation.
Examples:

1. C = {x: x is an integer, x > —3}


This is read as: "C is the set of elements x such that x is an integer greater
than -3.

2. D = {x: x is the capital city of the Philippines}

We should describe a certain property which all the elements x, in a set, have in common
so that we can know whether a particular thing belongs to the set.
We relate a member and a set using the symbol e. If an object x is an element of set A,
we write x e A. If an object z is not an element of set A, we write x A
e denotes "is an element of" or "is a member of" or "belongs to".
denotes "is not an element of" or "is not a member of" or "does not belongs to".

39
Example:

If A = then 1 e A and 2

EXERCISE:

A. Read the following questions. Choose the correct answer.


1. Which of the following set is shown with roster notation?

b. G = {letters in the English alphabet}


c. X = {red, blue, yellow}
d. None of the above.
2. Which of the following is true about set D listed below?

c. D is an infinite set.
d. All of the above
3. Which of the following is an infinite set?

a. {xe : x ž 2andxs 6 }
b. A = {letters in the English alphabet}
c. B = {Asia, Africa, North America, South America, Antarctica, Europe, Australia}
d. C = {integers}
4. Which of the following sets is equal to the set listed below?

a. xž2}
b. Y = {prime numbers less than 20}
C. Z = {odd numbers less than 20}
d. All of the above.
5. Which of the following is the correct set-builder notation for the set listed below?
{-5, -4, -3, -2, -1, o, 1,2, 3, ... }

d. All of the above.


B. Write each given set in the Set-Builder Form:
1. {2, 4, 6, 8, 10}
2. {2, 3, 5, 7, 11} 3. {January, June, July} 4.

40
5. {Tuesday, Thursday}
c. Write each of the following sets in the Roster form:
The set of first seven natural numbers.
2. The set of whole numbers less than 5.
3. The set of five numbers each of which is divisible by 3.
4. The set of whole numbers less than 20 and divisible by 3.
5. The set of integers greater than -2 and less than 4.

D. Using the items in B, write each set in a form of describing elements

5.2. Set Theory Symbols


Symbol Name Example Explanation
Set Collection of objects

c = {3,9}
Intersect An B = {3} Belong to both set A and set

Union Belon to set A or set B


Proper Subset A set that is contained in
CcB another set.
Subset A set that is contained in or
{1,3} c A equal to another set.
Not a Proper {1,3} A A set that is not contained
Subset in another set.
Superset Set B includes set C
Is a member 3 is element in set A
Is not a member 4 is not an element in Set A.

Other concepts of sets


The empty set or null set is the set that has no elements.
The cardinality or cardinal numbers of a set is the numbers of elements in a set.
Two sets are equivalent if they contain the same number of elements.
Two sets are equal if the contain the exact same elements although their order can be
different.
N, the set of natural numbers (non-negative integers) 0,1,2,
Z, the set of all integers
Q, the set of rational numbers, i.e. the ones of the form p q where p and q are integers
with q not equal to zero. R, the set of real numbers.
Example of using the symbols in sets

41
The set of natural numbers N contains the elements 1,2,3.
The set of integers Z contains all the natural numbers together with their negative and
zero: -3, -2, -1, O, 1, 2, 3
The set of rational numbers Q contains of numbers of the form a/b where a and b are
integers with b=0, for examples 1/3, 5,1, -22/7, 0/19, etc. Hence all integers are rational
numbers, but some rational numbers are not integers.
The set of all real numbers is denoted by R. The set of irrational number 3 contains of all
real numbers which are not rational, such as v'î,n, etc.
The set of even numbers E contains the elements 0, ±2, ±4, ±6, which are those of the form
2n for the same integer n. The set off odd numbers U is the set of integers which are not
even. Hence odd numbers are ±1, ±3, which can be written as 2n+1 for some integer n.

In Summary

N = {x e Zlx > 0}
E = {2nln e Z}

Q = {a/bla e Tb e N}

Example:

Describe the elements of each set.

2. Set B
3. Describe the subset and universal set shown below.

42
Answers:
In this example, we can describe the
sets this way.
Set Y is the universal set.
The set of vowels and the first five
consonants in alphabet.
Set X is a subset of Y.
The set of first five consonants in

EXERCISE:

A. If,

70} c =
{12, 74, 73 20, 24}
D {27, 26, 37, 36}

1. State whether true or false.


a.13 e c

d. 24 e c
e. 31 e D
2. Fill in the blanks.

c. 26 c

5.3. Subset

Subset is a set of whose elements are all members of another set.


The symbol "c", means "is a subset of".
The symbol "c", means "is a proper subset of".
Example:

43
W: The whole numbers: {0,1,2,3 ... }

Since all of the members of set A are members of set D, A is a subset of D.


Symbolically this is represented as A C D.
Note that A C D implies that n(A) n(D) (i.e. 3 6).

Note that A is also a proper subset of D since set D has members that do not belong to set A (A
D). Symbolically this is represented as A c D.
Note that A c D implies that n(A) < n(D) (i.e. 3 < 6).
Since some of the members of set C are NOT members of set D, C is NOT a subset of D.
Symbolically this is represented as C D.

Since all of the members of set A are members of set B, A is a subset of B. Symbolically this is
represented as A C B.
Although A C B, since there are no members of set B that are NOT members of set A (A
= B), A is NOT a proper subset of B.

Any set is considered to be a subset of itself.


No set is a proper subset of itself.
The empty set is a subset of every set.
The empty set is a proper subset of every set except for the empty set.
5.4. Solving Problems using the Concepts of Set Theory including the methods of Venn
Diagrams

Set theory has its own notations and symbols that can seem unusual for many. In this
tutorial, we look at some solved examples to understand how set theory works and the kind
of problems it can be used to solve.
For example
Set of natural numbers
Set of whole numbers
Each object is called an element of the set.

44
The set that contains all the elements of a given collection is called the universal set and is
represented by the symbol pronounced as 'mu'.

For two sets A and B, n(AuB) is the number of elements present

in either of the sets A or B.

n(AnB) is the number of elements present in both the sets A and B.


n(AuB) = n(A) + (n(B) — n(AnB)
For three sets A, B and C, n(AuBuC) = n(A) + n(B) + n(C) - n(AnB) - n(BnC) -
n(CnA) + n(AnBnC)

Consider the following example:

1. In a class of 100 students, 35 like Math and 45 like English. 10 like both. How many like
either of them and how many like neither?
Solution:
Total number of students, n(u) = 100

Number of Math students, n(M) = 35

Number of English students, n(E) = 45

Number of students who like both, n(EnM) = 10


Number of students who like either of them,
n(EuM) = n(E) + n(M) - n(EnM) 45+35-10 = 70
Number of students who like neither = n(u) — n(EuM) = 100 - 70 = 30
Using Venn Diagram, we can easily solve the problem.

10 Math
35

As it is said, one picture is worth a thousand words. One Venn diagram can help solve
the problem faster and save time. This is especially true when more than two categories are
involved in the problem.

45
2. There are 30 students in a class. Among them, 8 students are learning both English and
French. A total of 18 students are learning English. If every student is learning at least
one language, how many students are learning French in total?

Solution:
The Venn diagram for this problem looks like this.

Every student is learning at least one language. Hence there is no one who fall in the
category 'neither'.

So in this case, n(EUF) = n(u).


It is mentioned in the problem that a total of 18 are learning English. This DOES NOT mean
that 18 are learning ONLY English. Only when the word 'only' is mentioned in the problem
should we consider it so.

Now, 18 are learning English and 8 are learning both. This means that 18 — 8 = 10 are learning
ONLY English.

no = 30, n(E) = 10 n(EUF) =


n(E) + n(F) - n(EnF)
30 = 18+ n(F) -8 n(F)

20
Therefore, total number of students learning French = 20.
Note: The question was only about the total number of students learning French and not
about those learning ONLY French, which would have been a different answer, 12.
Finally, the Venn diagram looks like this.

46
3. Among a group of students, 50 played basketball, 50 played softball and 40 played volley
ball. 15 played both basketball and softball, 20 played both softball and volley ball, 15
played basketball and volley ball and 10 played all three. If every student played at least
one game, find the number of students and how many played only basketball, only softball
and only volley ball? Solution:
n(B) = 50, n(S) = 50, n(V) = 40 n(BnS)
= 15
n(SnV) = 20
n(BnV) = 15
n(BnSnV) 10
No. of students who played at least one game n(BuSuV) = n(B) + n(S) +
n(V) - n(BnS) - - n(BnV) + n(BnSnV)
50 + 50 + 40-15-20-15 + 10
Total number of students - 100
Let a denote the number of people who played basketball and volleyball only.
Let b denote the number of people who played basketball and softball only.
Let c denote the number of people who played softball and volleyball only.
Let d denote the number of people who played all three games.
Accordingly, d = n (BnSnV) = 10
Now, n(BnV) = a + d = 15 n(BnS)
= b + d 15 n(SnV) = c + d = 20
Therefore, a = 15 — 10 = 5 [basketball and volleyball
only] b = 15 — 10 = 5 [basketball and softball only] c =
20 — 10 = 10 [softball and volleyball only]

47
No. of students who played only basketball = n(B) — [a + b + d] = 50 — (5 + 5 + 10) = 30
No. of students who played only softball = n(S) — [b + c + d) = 50 — (5 + 10 + 10) = 25
No. of students who played only volley ball = n(V) — [a + c + d] = 40 — (10 + 5 + 10) = 15

Alternatively, we can solve it faster with the help of a Venn diagram.


The Venn diagram for the given information looks like this.

50
Basketball

50

Subtracting the values in the intersections from the individual values gives us the number of
students who played only one game.

EXERCISE:
Read the following problem. Solve the problems using the concepts of Set
Theory including the methods of Venn Diagrams.

There are 35 students in art class and 57 students in dance class. Find the
number of students who are either in art class or in dance class.
a. When two classes meet at different hours and 12 students are enrolled
in both activities.
b. When two classes meet at the same hour.
2. In a group of 100 persons, 72 people can speak English and 43 can speak
French. How many can speak English only? How many can speak French only
and how many can speak both English and French?

48
3. In a competition, a school awarded medals in different categories. 36 medals in
dance, 12 medals in dramatics and 18 medals in music. If these medals went to
a total of 45 persons and only 4 persons got medals in all the three categories,
how many received medals in exactly two of these categories?

CHAPTER TEST:
A.Read the following statements. Give what is being asked.
1. Write the following sets in the roster form.
A = The set of all even numbers less than 12
B = The set of all prime numbers greater than 1 but less than 29
C = The set of integers lying between -2 and 2
D = The set of letters in the word LOYAL E = The set of vowels in the word
CHOICE
2. Write each of the following in set builder form.
A {5, 10, 15, 20}

3. Write the cardinal number for each of the following.


X = The set of months in a year
Y = The set of letters in the word INTELLIGENT
Z = The set of prime numbers from 2 to 11
P = {x : x is an even prime number}
Q = {x : x is a quadrilateral having 5 sides}

B.Describe the elements of each set.

1. SetA 2. Set B

3.Set C 4. Set D

49
C. Describe the subset and universal set shown below

D.Use the diagram below to answer the


questions that follow.
What elements are found in the union of A and
B?
2. What is the cardinality of the union of A and
B?
3. What elements are found in the intersection
of A and B?
4. What is the cardinality of the intersection of
A and B?
5. What are the elements in A'? of B'?

E. Solve the following problem.

In a group, there were 115 people whose proofs of identity were being verified.
Some had passport, some had voter id and some had both. If 65 had passport
and 30 had both, how many had voter id only and not passport?

2. Among a group of people, 40% liked red, 30% liked blue and 30% liked green. 7%
liked both red and green, 5% liked both red and blue, 10% liked both green and

50
blue. If 86% of them liked at least one colour, what percentage of people liked all
three?
Reference:
http ://www.learnalberta.ca/content/memg/Division03/Subset% 200f%
20a%20Set/index.html http://pfister.ee.duke.edu/courses/ecen601/notes chl.pdf
https://www.mathgoodies.com/lessons/sets/practice unit15 https://www.math-only-
math.com/worksheet-on-sets-in-roster-form.html
https://www.math-only-math.com/worksheet-on-sets-in-set-builder-form.html
https://www.math-only-math.com/word-problems-on-sets.html

51
CHAPTER 6
Basíc Set ofOperatíons
Introduction
A Set is an unordered collection of objects, known as elements or members of the set.
An element 'a' belong to a set A can be written as 'a e A', 'a A' denotes that a is not an
element of the set A. In this chapter, we will be tackled the different basic sets of operations.
It includes union, intersection, difference and disjoint.
This chapter presents the different basic operations on sets namely union,
intersection, and complement. It will also tackle the subtraction or difference and the
mutually exclusive or disjoint set.

the
numbers.

Lecture Notes
6.1. Set Operation
When two or more sets combine together to form one set under the given conditions,
then operations on sets are carried out

61.1. Union

The union of two sets is a set containing all elements that are in A or in B (possibly
both). We denote the union of A and B by AuB.
The set defined as

52
AUB is shaded
Example:
If A = and B = {4,5}, then A U B =
2. If A = {1,2,3} and B = {1,2,4,5}, then A u B = {1,2,3,4,5} (Note that elements are not
repeated in a set.)
3. Given the Venn Diagram

The union is A UB = {a, b, c, d, e, f, g, h, i}


4. Let A be the set of positive real numbers and let B the set of negative real numbers.
Then AUB, consists the set of all real numbers except zero.

Take note:
• A u Bis always the same as B UA (A UB = B U A )
• Both A and B are always subsets of A u B where A c (A u B)and B c (A u B).
6.12. Intersection
The intersection of two sets is a set common elements in A and in B. They are the
elements that are belong to the to A and also belong to B. We denote the intersection of A
and B by A n B.
The set defined as

An B = {xlx e A n xe B}
The Venn Diagram below shows the intersection of A and B

53
61.3. Complement
The complement of a set A, denoted by A C or 14, is the set of all elements that are in
the universal set S but are not in A. The complement of A may be denoted as

In Figure below, A is shown by the shaded area using a Venn diagram. s

Example:
Let the universal set S bet the first 10 numbers and let V the first 5 even numbers. Then
the
2. Let the universal set S the English Alphabet, and let V the vowels. Therefore, the v c —
{consonant letters in alphabet}

61.4. Difference or Subtraction


The difference (subtraction) is defined as follows. The set A—B consists of elements
that are in A but not in B. The difference of A and B is also called the complement of B with
respect to A. Note that A-B=AnB C • It is denoted by

54
The Figure below shows A-B.

s
A B

Example:
1. If and then

61.5. Mutually Exclusive or Disjoint


Two sets A and B are mutually exclusive or disjoint if they do not have any shared
elements; i.e., their intersection is the empty set, AnB=Ø. More generally, several sets are
called disjoint if they are pairwise disjoint, i.e., no two of them share a common element.

The Venn Diagram shows disjoint sets

Sets A, B, and C are disjoint sets.


Example

1 If set {1,2,3,6} and set {4,7,8}, therefore it is a disjoint sets(AnB=Ø).


2. If set consonants letter and set B = vowel letters, therefore, AnB=Ø

EXERCISE:

Alf c, d, e}, V={d, e, f, g}, g, h, i}, find the following:


1.
2. V I-J W

55
4. U U (V U W)
5.

10. W'

B. Find the union, intersection and the difference (A - B) of the following pairs of sets.
1. a = e N, x is a factor of 12} b = {x I x e N,
x is a multiple of 2, x < 12}

6.2. Algebra of Set This section contains many results concerning the
properties of the set operations.

62.1. Theorem 1.

Let A, B, and C be subsets of some universal set U. Then


AnBCA and ACAuB.
If ACB, then AnCCBnC and AUCCBUC.

6.22. Theorem 2.

Let A, B, and C be subsets of some universal set U. Then all of the following equalities
hold.

• Proporties of the Empty Set AnØ=Ø AnU=A and the Universal Set AUØ=A AUU=IJ.

• Idempotent Laws AnA=A , AUA=A

• Commutative Laws AnB=BnA, AUB=BUA.

• Associative Laws (AnB)nC=An(BnC)

56
• Distributive Laws An(BUC)=(AnB)U(AnC)

We will prove that AnB=BnA. Let xeAnB. Then xeAnB if and only

if xeA and xeB. (Eq. 1)

However, we know that if P and Q are statements, then PwedgeQ is logically


equivalent to QAP. Consequently, we can conclude that xeA and xeB if and only if XEB
and xeA (Eq. 2)

Now we know that

XEB and xeA if and only if xeBnA.(Eq. 3)

This means that we can use Eq. 1, Eq. 2 and Eq. 3 to conclude that

xeAnBxeAnB if and only if xeBnAxeBnA, and, hence, we have

proved that AnB=BnA.

62.3. Important Properties of Set Complements

The next theorem states some basic properties of complements and the important
relations dealing with complements of unions and complements of intersections. Two
relationships in the next theorem are known as De Morgan's Laws for sets and are closely
related to De Morgan's Laws for statements.

Theorem 3

Let A, B, and C be subsets of some universal set U. Then the following are true:

Basic Properties
A-B=AnBC
Empty Set and Universal Set
ØC =U and U C =Ø

Subsets and Complements ACBACB if and only if B C CAC

57
Proof:

Let A and B be subsets of some universal set U. We will prove that by proving
C C
that an element is in (AUB)c if and only if it is A nB So let x be in the universal set U. Then
x e (A u if and only of x A u B Eq.3
And
x A u B if and only if x A and x B Eq.4
Combining Eq. 3 and Eq. 4, we see that

x e (A u BY if and only of x A and x B Eq.5


In addition, we know that

A and x B if and only if x e A C and x e B C Eq. 6


C C
And this is true if and only if x e A u B So we can use Eq. 5 and Eq. 6 to conclude that

x e (A u if and only of x E AC n BC.


And, hence, that (A u BY = A C n B C

EXERCISE:
Let A be a subset of some universal set UU. Prove each of the following (from Theorem
3):

(c) Øc=U
(d) Uc=Ø
2. Let A, B, and C be subsets of some universal set U. As part of Theorem 2, we proved
one of the distributive laws. Prove the other one. That is, prove that

3. Let A, B, and C be subsets of some universal set U. As part of Theorem 3, we proved


one of De Morgan's Laws. Prove the other one. That is, prove that

CHAPTER TEST:

1. D {8, 9, 10, 11}, find

58
2. If A = {4, 6, 8, 10, 12} B {8, 10, 12, 14} C = {12, 14, 16} D = {16, 18}, find

3. If A {4, 7, 10, 13, 16, 19, 22} B = {5, 9, 13, 17, 20} c = {3, 5, 7, 9, 11, 13, 15, 17} D = {6,
11, 16, 21} , then find

(f) C - D

4. If A and B are two sets such that A c B, then what is AUB?


5. Find the union, intersection and the difference (A - B) of the following pairs of
sets.

The set of all letters of the word FEAST


B = The set of all letters of the word TASTE

59
The set of all even numbers less than 12
B = The set of all odd numbers less than 11
= {x : x e l, -2 < x < 2}

6. Let X - Z {5, 6, 7, 8}, find

7. {l, 2, 3, 4, 5, 6, 7} and A {1, 2, 3, 4, 5} B -- {2, 5, 7} show that

8. Let P = {a, b, c, d} Q = {b, d, f} R {a, c, e} verify that

(a) (P u Q) U R = P U (Q U R)

9. If the universal set is given by S = {1,2}, C-- {1,5,6} are


three sets, find the following sets:

(a) AUC
(b) AUBAUB
(c) AnBAnB
d.

f.
10. Let A, B, and C be subsets of some universal set U. Prove or disprove each of the
following:

11. Determine which properties from Theorems 2 and 3. Justify each of the last three
steps in the following outline of the proof that (AUB)-C=(A—C)U(B-C).

Theorem 3

60
Commutative Property

cc nA) U ( cc nB)

References:

https://www.math-only-math.com/operations-on-sets.html

https://people.cs.pitt.edu/—milos/courses/c s441/1ectures/Cl ass7 .pdf

https://www.cs.odu.edu/—toida/nerzic/level-a/set/set operations.html https://www.math-only-

math.com/practice-test-on-operations-on-sets.html

https://math.libretexts.org/Bookshelves/Mathematical Logic and Proof/Book%3A Mathematic


al Reasoning Writing and Proof (Sundstrom)/5%3A Set Theory/5.3%3A Properties of Set
Operations

61
CHAPTER 7
Sets ofNumbers
Introduction:
A set in mathematics is a collection of well-defined and distinct objects, considered as
an object in its own right. Sets are one of the most fundamental concepts in mathematics.
Developed at the end of the 19th century, set theory is now a ubiquitous part of
mathematics, and can be used as a foundation from which nearly all of mathematics can be
derived. (Georg Cantor, the founder of set theory).
This chapter will discuss the sets with regards on proving. It will also use the method of
Venn diagram and truth tables to prove the sets.

Lecture Notes
7.1 SET
A set is a gathering together into a whole of definite, distinct objects of our
perception and of our thought — which are called elements of the set. The elements or
members of a set can be anything: numbers, people, letters of the alphabet, other sets, and
so on. Sets are conventionally denoted with capital letters. Sets A and B are equal if and only
if they have precisely the same elements.

There are two ways of describing, or specifying the members of, a set. One way is by
intentional definition, using a rule or semantic description:
A is the set whose members are the first four positive
integers. B is the set of colors of the French flag.

The second way is by extension — that is, listing each member of the set. An
extensional definition is denoted by enclosing the list of members in curly brackets:

D = {blue, white, red}.


Every element of a set must be unique; no two members may be identical. (A multi-
set is a generalized concept of a set that relaxes this criterion.) All set operations preserve
this property. The order in which the elements of a set or multi-set are listed is irrelevant
(unlike for a sequence or tuple). Combining these two ideas into an example:
{6, 77} {77, 6, 6,

62
The extensional specification means merely that each of the elements listed is a member of
the set.
For sets with many elements, the enumeration of members can be abbreviated. For
instance, the set of the first thousand positive integers may be specified extensionally as:

{7, 2, 3, 7000},

Where the ellipsis (" indicates that the list continues in the obvious way. Ellipses may also be
used where sets have infinitely many members. Thus the set of positive even numbers can
be written as {2, 4, 6, 8,

7.2 Basic Operations of Sets


72.1 Unions

Two sets can be "added" together. The union of A and B, denoted by A u B, is the set
of all things which are members of either A or B.

Examples:
{1, 2} u {red, white} 2, red, white}.
{1, 2, green} u {red, white, green} 2, red, white, green}.

Some basic properties of unions:

A U B = B UA.

AC B if and only if A U B = B

Learn how to represent the union of sets using Venn diagram. The union set
operations can be visualized from the diagrammatic representation of sets.
The rectangular region represents the universal set U and the circular regions the
subsets A and B. The shaded portion represents the set name below the diagram.

Let A and B be the two sets. The union of A and B is the set of all those elements
which belong either to A orto B or both A and B.

Now we will use the notation A U B (which is read as 'A union B') to denote the union
of set A and set B.

63
Thus, e B}.
Clearly, x E A U B

For Examples:

l. If A {2, 5, 7} and B = {1, 2, 5, 8}. Find AUB using Venn diagram.


Solution:

According to the given question we know, A = {2, 5, 7} and B = {1, 2, 5, 8} Now let's
draw the Venn diagram to find A union B.

Therefore, from the Venn diagram we get A U B =


2. From the adjoining figure find A union B.

01235
AUB= 89

64
Solution:
According to the adjoining figure we get;

Set A = {0, 1, 3, 5, 8}
Set B {2, 5, 8, 9}

Therefore, A union B is the set of elements which in set A or in set B or in both.


Thus, A U B = {0, 1, 2, 3, 5, 8, 9}

7.22 Intersections

A new set can also be constructed by determining which members two sets have "in
common". The intersection of A and B, denoted by A n B, is the set of all things which are
members of both A and B. If A n B = Ø, then A and B are said to be disjoint.

Examples:
{1, 2} n {red, white} = Ø
{1, 2, green} n {red, white, green} = {green}

Some basic properties of intersections:


An B = B n A.

AC B if and only if A n B = A
Learn how to represent the intersection of sets using Venn diagram. The intersection
set operations can be visualized from the diagrammatic representation of sets.

The rectangular region represents the universal set U and the circular regions the
subsets A and B. The shaded portion represents the set name below the diagram.

Let A and B be the two sets. The intersection of A and B is the set of all those
elements which belong to both A and B.

Now we will use the notation A n B (which is read as 'A intersection B') to denote the
intersection of set A and set B.

Thus, A n B = {x : x e Aand x e B}.


Clearly, x e A n B x E A and x
For Examples:
l) If A = {1, 2, 3, 4, 5} and B = {1, 3, 9, 12}. Find A n B using Venn diagram.
Solution:

65
According to the given question we know, A = {1, 2, 3, 4, 5} and B = {1, 3, 9, 12}
Now let's draw the Venn diagram to find A intersection B.

Therefore, from the Venn diagram we get A n B

2) From the adjoining figure find A intersection B.

m
Solution:

According to the adjoining figure we get;

Set A = {m, p, q, r, s, t, u, v}
Set B = {m, n, o, p, q, i, j, k, g}

Therefore, A intersection B is the set of elements which belong to both set A and set B.

Thus, A n B = {p, q, m}
72.3. Complements

Two sets can also be "subtracted". The relative complement of B in A (also called the
set-theoretic difference of A and B), denoted by A \ B (or A - B), is the set of all elements
which are members of A but not members of B. Note that it is valid to "subtract" members of
a set that are not in the set, such as removing the element green from the set {1, 2, 3}; doing
so has no effect.

66
In certain settings all sets under discussion are considered to be subsets of a given
universal set U. In such cases, U \ A is called the absolute complement or simply
complements of A, and is denoted by A'

Examples:
{1, 2} \ {red, white} = {1, 2}.
{1, 2, green} \ {red, white, green} -

If U is the set of integers, E is the set of even integers, and O is the set of odd integers,
then U \ E E' O.

Some basic properties of complements:

A \ B B \ A for A B.

U' = Ø and Ø'

An extension of the complement is the symmetric difference, defined for sets A, B as :

A AB = (A/B) U (BIA)
For example, the symmetric difference of {7,8,9,10} and {9,10,11,12} is the set {7,8,11,12}.

The complement of a set using Venn diagram is a


subset of U. Let U be the universal set and let A be a set
such that A c U. Then, the complement of A with respect to
U is denoted by A' or ACC or U — A or A and is defined the
set of all those elements of U which are not in A.

Thus, A l {x U x A}.

Clearly, x xA

Com lement of A or A'

67
(A — B) is also called the complement of B relative to A. From the definition it is clear
that the complement of the whole set in a set is the null set; for U' = U — U = Ø again Ø' = U
Ø = U also = U —A = U — (U —A) = A. If the set of real numbers be the universal set, then the
set of rational numbers and the set of irrational numbers are
complements of each other.

For Examples:
1. Let the set of natural numbers N be
the universal set and let A = {2, 4, 6, 8, Then A' = {1, 3, 5,
2. If U = {l, 2, 3, 4, 5, 6, 7, 8, 9} and A = {1, 3, 5, 7, 9} then A'
A' {2, 4, 6, 8}
Com lement of A or
A

6
om lement

Exercises:

1. Let
Give the sets resulting from:
a) AnBAnB
b) CUACUA
c) CnDCnD

2. Let and C— the set of vowels. Calculate:


a) AUBUC.AUBUC.
b) AnB.AnB.
c) c c

68
7.3 TYPE OF SETS

73.1. Empty Set or Null Set:


A set which does not contain any element is called an empty set, or the null set or
the void set and it is denoted by Ø and is read as phi. In roster form, Ø is denoted by 0. An
empty set is a finite set, since the number of elements in an empty set is finite, i.e., 0.

For example:
(a) The set of whole numbers less than 0.
(b) Clearly there is no whole number less than 0.
Therefore, it is an empty set.

Let A = {x : 2 < x < 3, x is a natural number} Here A is an empty set because


there is no natural number between 2 and 3.
Let B = {x : x is a composite number less than 4}. Here B is an empty set
because there is no composite number less than 4. Note:
Ø {0} has no element.
{0} is a set which has one element 0.
The cardinal number of an empty set, i.e., n(Ø) = 0

7.32. Singleton Set:


A set which contains only one element is called a singleton set.

Example:
A = {x : x is neither prime nor composite}lt is a singleton set containing one element,

B = {x : x is a whole number, x < 1}This set contains only one element 0 and is a
singleton set.

Let A = {x : x e N and x 2 = 4} Here A is a singleton set because there is only one


element 2 whose square is 4.
Let B = {x : x is a even prime number}Here B is a singleton set because there is only
one prime number which is even, i.e., 2.

7.3.3. Finite Set:


A set which contains a definite number of elements is called a finite set. Empty set is
also called a finite set.

For example:
The set of all colors in the rainbow.

69
P = {2, 3, 5, 7, 11, 13, 17, 97}

7.3.4. Infinite Set:


The set whose elements cannot be listed, i.e., set containing never-ending elements is
called an infinite set.

For example:
Set of all points in a plane

Set of all prime numbers

Note:
All infinite sets cannot be expressed in roster form.

For example:
The set of real numbers since the elements of this set do not follow any particular
pattern.

73.5 Cardinal Number of a Set:


The number of distinct elements in a given set A is called the cardinal number of A. It
is denoted by n(A). For example:

Therefore, n(A) = 4

B = set of letters in the word ALGEBRA

Therefore, n(B) = 6

73.6. Equivalent Sets:


Two sets A and B are said to be equivalent if their cardinal number is same, i.e.,
n(A)= n(B). The symbol for denoting an equivalent set is For example:
A {1, 2, 3} Here n(A) = 3
B = {p, q, r} Here n(B) = 3 Therefore, A B

73.7. Equal sets:


Two sets A and B are said to be equal if they contain the same elements. Every
element of A is an element of B and every element of B is an element of A.

For example:

70
Therefore, A = B
The various types of sets and their definitions are explained above with the help of examples.

73.8. Subsets
If every member of set A is also a member of set B, then A is said to be a subset of B,
written A C B (also pronounced A is contained in B). Equivalently, we can write B 2 A, read as
B is a superset of A, B includes A, or B contains A. The relationship between sets established
by C is called inclusion or containment.

If A is a subset of, but not equal to, B, then A is called a proper subset of B, written A g
B (A is a proper subset of B) or B 2 A (B is a proper superset of A).
Examples:
The set of all men is a proper subset of the set of all people.

The empty set is a subset of every set and every set is a subset of itself:

An obvious but useful identity, which can often be used to show that two seemingly
different sets are equal:

A = B if and only if A C B and B C A.

A partition of a set S is a set of nonempty subsets of S such that every element x in S is


in exactly one of these subsets.

For example:
1. Let A {2, 4, 6}

Here A is a subset of B. Since, all the elements of set A are contained in set B. But B is
not the subset of A, since, all the elements of set B are not contained in set A.

7.3.9. Proper Subset:

If A and B are two sets, then A is called the proper subset of B if A C B but B A i.e. A B.
The symbol 'c' is used to denote proper subset. Symbolically, we write A c B.
For example;
1. A = {1, 2, 3, 4} Here n(A) 4

71
B {l, 2, 3, 4, 5} Here n(B) = 5

We observe that, all the elements of A are present in B but the element '5' of B is not
present in A. So, we say that A is a proper subset of B. Symbolically; we write it as A c B

7.3.10.Power Sets

The collection of all subsets of set A is called the power set of A. It is denoted by P(A).
In P(A), every element is a set.

For example:

If A = {p, q} then all the subsets of A will be


P(A) = {Ø, {p}, {q}, {p, q}}
Number of elements of P(A) = n[P(A)] = 4 = 22

In general, n[P(A)] = 2m where m is the number of elements in set A.

73.11 Universal Set

A set which contains all the elements of other given sets is called a universal set The
symbol for denoting a universal set is U or

For example;
1. If A = {l, 2, 3} B {2, 3, 4} C = {3, 5, 7} then U {l, 2, 3, 4, 5, 7} Here A U, B c U, cc U
and U a A, U B, U 2 C
2. If P is a set of all whole numbers and Q is a set of all negative numbers then the universal
set is a set of all integers.

3. If A = {a, b, c} B = {d, e} C = {f, g, h, i} then U = {a, b, c, d, e, f, g, h, i} can be


taken as universal set.

Exercises:
A. State whether the following set is infinite or finite, give support to your answer.

a. Set of integers
b. {Multiples of 5}
c. {Fractions between 1 and 2}
d. {Numbers of people in India}
e. Set of trees in the world

72
f. Set of prime numbers
g. Set of leaves on a tree
h. Set of children in all the schools of Delhi} Set of seven natural numbers
j.
k. {-12, -9, -6, -3, o, 3, 6,
{Number of points in a line segment 4 cm long}.
B. Write the universal set for the following.

(c) Prime numbers less than 10, even numbers less than 10, multiples of 3 less than 10.

C. Identify the following types of sets.


a. A set containing no elements is called the empty set or null set or
void set.
b. A set is said to be an infinite set if the number of elements in the set
is not finite.
c. Two sets A and B are said to be equal if they contain exactly the same
elements, regardless of order.
d. A set which contains all the elements of other given sets.
e. A set which contains only one element.

CHAPTER TEST

A. State whether true or false.


a. 13 ec

d. 24 C
e. 31 e D
f. 36 e D
g. 20 e c
h.

B. Are two sets A and B equal? Give reasons to support your


answer.

{x : x is a letter in the word SEAT}


{x : x is a letter in the word TASTE}

73
{2, 6, 10, 14}
{6, 2, 14, 16}

{x : x is a positive odd integer x 9}

B = {x : x > 15 and x < 5}

c. — {2, 4}. Fill in the blanks


by c or C to make the resulting statements true.

D. Write the universal set for the following.

(c). Prime numbers less than 10, even numbers less than 10, multiples of 3 less than 1

E State whether the following set is infinite or finite, give support to your
answer.
m. Set of integers
n. {Multiples of 5}
o. {Fractions between 1 and 2}
p. {Numbers of people in India}
q. Set of trees in the world
r. Set of prime numbers
s. Set of leaves on a tree
t. Set of children in all the schools of Delhi}
u. Set of seven natural numbers

74
w. {-12, -9, -6, -3, o, 3, 6,

x. {Number of points in a line segment 4 cm long}.

F If A = {2, 3, 4, 5}, B d, e, f} and C = {4, 5, 6, 7}; Find:


a. AUB
b. AUC

c.
d.
e.

G. If: A = Set of natural numbers,


B = Set of prime numbers and
C = Set of even prime numbers.
Draw Venn-diagram showing the relationship among the given sets A, B and C.

References:

http://home.ku.edu.tr/-amostafazadeh/math107/F2012/Handouts/Sets%20from
%20Wikipedi a 2012 09 18.pdf https://www.math-only-math.com/union-of-sets-using-venn-
diagram.html https://www.math-only-math.comffintersection-of-sets-using-\/enn-
diagram.html https://www.math-only-math.com/complement-of-a-set-using-Venn-

https://www.math-only-math.com/types-of-sets.html
https://www.math-only-math.com/subset.html
https://www.math-only-math.com/worksheet-on-finite-and-infinite-sets.html
https://www.math-only-math.com/worksheet-on-set.html https://www.math-only-
math.com/practice-test-on-sets-and-subsets.html
https://www.math-only-math.com/worksheet-on-union-and-intersection-of-sets.html

75
CHAPTER 8
functíons
Introduction:
We can define functions in a way that depends only on the concept of set. Function in
mathematics, an expression, rule, or law that defines a relationship between one variable
(the independent variable) and another variable (the dependent variable). Peter Dirichlet
7873
This chapter deals functions. It starts with the concept of function by its types. This
chapter also has exercises after each lessons and a chapter test at the end.

Lecture Notes
8.1. FUNCTIONS

If A and B are two non-empty sets, then a relation 'f' from set A to set B is said to be a
function.

• If every element of set A is associated with unique element of set B.

• The function 'f' from A to B is denoted by f: A B.

• If f is a function from A to B and x e A, then f(x) e B where f(x) is called the image of x
under f and x is called the pre image of f(x) under 'f'.

A Function F from a set A to a set B is a relation with domain-A and co-domain B that
satisfies the following two properties:
For every element x in A, there is an element y in B such that (x, y) e F.
For all elements x in A and y and z in B,
If (x, y) e F and (x, z) e F, then

Sets and functions are two of the most basic ideas in mathematics, and we'll need to
know what they are to discuss some things about categories rigorously. Sets are imaginary

76
bags we put things into. For example, you can take a dog, a cat, and a shoe, put them in an
imaginary bag, and now you have a set consisting of {dog, cat, shoe}. The members of the set
—dog, cat, and shoe—are called the elements of the set. While Functions are maps between
sets that meet a few rules. For example, Think of sets as countries, and the elements of the
sets as cities in that country. A map between the two sets is a map telling you how to go
from the cities in one country to the cities in the other country.
In other words, Set is a well-defined collection of distinct objects of our perception or
of our thought, to be conceived as a whole' and Functions is a relation from a set of inputs to
a set of possible outputs where each input is related to exactly one output.

Notation:
If A and B are sets and F is a function from A to B, then given any element x in A
unique element in B that is related to x by F is denoted F(x), which is read "F of x

For f to be a mapping from A to B:

• Every element of A must have image in B. Adjoining figure does not


represent a mapping since the element d in set A is not associated
with any element of set B.

• No element of A must have more than one image. Adjoining figure


does not represent a mapping since element b in set A is
associated with two elements d, f of set B.

• Different elements of A can have the same image in B. Adjoining


figure represents a mapping.

The domain of a function f(x) is the set of all values for which the function is defined,
and the range of the function is the set of all values that following takes.
Examples:

• Consider the function shown in the diagram.

77
Here, the domain is the set {A,B,C,E}{A,B,C,E} . DD is not in the domain, since the function is
not defined for DD.

The range is the set {1,3, 4}{1,3,4} . 2 is not in the range, since there is no letter in the domain
that gets mapped to 2.

• Consider the relation (2, 10)} .

Here, the relation is given as a set of ordered pairs. The domain is the set of xx coordinates,
{0,1,2}{0,1,2} , and the range is the set of y -coordinates, {7,8,9,10}{7,8,9,10} Note that the
domain elements 11 and 22 are associated with more than one range elements, so this is
not a function.

Exercises:

1. Determine whether the relation represents a function.

a) {(a,b),(c,d),(a,c)}
b) {(a,b),(b,c),(c,c)}

2. Determine whether the relation represents y as a function ofx.

3. What is the difference between relation and function?

4. How do we determine the domain of a function defined by an equation?

8.2. TYPES OF FUNCTIONS

82.1. One to One Function


A function f: A B is One to One if for each element of A there
is a distinct element of B. It is also known as Injective. Consider if al
e A and a2 e B, f is defined as f: A B such that f (al) = f

8.2.2.Many to One Function

78
It is a function which maps two or more elements of A to the same element of set B.
Two or more elements of A have the same image in B.

82.3. Onto Function


If there exists a function for which every element of set B
there is (are) pre-image(s) in set A, it is Onto Function. Onto is also
referred as Surjective Function.

82.4. Into — function


If there exists at least one element in the co-domain which is not an image of any
element in the domain then the function will be Into function.
Example:
(Q) Let A = {x : 1 < x < 1} = B be a mapping f : A B, find the nature of the given function (P) F(x) = x
fix) = Ill
Solution for x = 1 & -1

Hence it is many one the Range of f(x) from [-1, 1] is [0,1] which is
not equal to co-domain. Hence it is into function.

-1 1

f (x) = Il l EXERCISES:

1. How can you determine if a relation is a one-to-one function?

2. Determine whether the relationship given in the mapping diagram is a function.

a)

79
Input Output

Input Out ut

10
15
c)

CHAPTER TEST

A.

80
B. LetA = {1, 2, 3} B = {3, 6, 9, 12, 15}
Draw the arrow diagram to represent the rule f(x) = from A to B.

C. Find the domain and range of each of the following functions.

(c) f (x) = x, x e R

References:

https://www.britannica.com/science/function-mathematics https://www.math-only-
math.com/functions-or-mapping.html
https://www.lesswrong.com/posts/4nqf7c4qnpa8mv7Yk/sets-and-functions
https://mathinsight.org/definition/function#:-:text=A%20technical%20definition%200f
%20a.r elated9620to%20exactly%200ne%200utput.&text=We%20can%20write%20the
%20statement.

https://byjus.com/jee/functions-and-its-types/
https://www.math-only-math.com/worksheet-on-functions-or-mapping.html
https://www.varsitytutors.com/hotmath/hotmath help/topics/domain-and-

set.

81
CHAPTER 9
Proact Sets andGraphs offunctíons
Introduction:
In 1943 crucial breakthrough were made by Norbert Wiener (1894-1964), a young
American who had recently received hid Ph. D. from Harvard and the German mathematician
Felix Hausdorff (1868-1942). Both gave definitions showing that an ordered pair can defined as
a certain type of set, but both definitions were somewhat awkward.
This chapter will tackle the concepts of ordered pairs, product sets and coordinate
diagram or graph of a functions.

Lecture Notes

9.1. Ordered Pair


An ordered pair is a composition of the x coordinate (abscissa) and the y coordinate
(ordinate), having two values written in a fixed order within parentheses.
It helps to locate a point on the Cartesian plane for better visual comprehension.
The numeric values in an ordered pair can be integers or fractions.
In the set theory, we learnt to write a set in different forms, we also learnt about
different types of sets and studied operations on sets and Venn diagrams. Also in co-ordinate
system we learnt about an ordered pair.

Given elements a and b, the symbol (a, b) denotes the ordered pair consisting of a and b
together with the specification that a is the first element of the pair and b is the second
element. Two ordered pairs (a, b) and (c, d) are equal if, and only if a-c and b=d. Symbolically,
(a, b) = (c, d) means that a-c and
The ordered pair (2, 5) is not equal to ordered pair (3, 2) i.e., (2, 5) (5, 2). Thus, in a
pair, the order of elements is important. An ordered pair consists of two elements that are
written in the fixed order.

For Examples:

82
1. Two ordered pairs (a, b) and (c, d) are equal if a = c and b = d, i.e., (a, b) = (c
d). Find the values of x and y, if (2x - 3, y + 1) -
Solution:
By equality of ordered pairs, we have - 3 = x + 5 and y + 1 = 7 -
x = 5 + 3 x = 8 and y = 7 - I =6
2.Given (3a, 3) =

Solution:

Then, 3a = 5a - 4 and 3 = b + 1
5a - 3a = 4 and b = 3 - 1 2a
= 4 and b = 2

EXERCISES:
1. Suppose the ordered pairs y-4) and 3y-2). Find x and y.
2. Given (x - 3, y + 2) = (4, 5), find x and y.
3. Given (2x-10, 3y-7)= (8,8), find x and y.
4.Suppose the ordered pairs (X+Y, 5) and (7, x-y) are equal, find x and y.

9.2. Product Set


The Cartesian products of sets mean the product of two non-empty sets in an ordered
way. Or, in other words the collection of all ordered pairs obtained by the product of two
nonempty sets. An ordered pair means that two elements are taken from each set.

If A and B are two non-empty sets, then their Cartesian product A x B is the set of all
ordered pair of elements from A and B.

Suppose, if A and B are two non-empty sets, then the Cartesian product of two sets, A and
set B is the set of all ordered pairs (a, b) such that a e A and b e B which is denoted as

For Examples:

- {7, 8} and B {2, 4, 6}, find A x B.


Solution:

83
The 6 ordered pairs thus formed can represent the position of points in a plane, if a and B are
subsets of a set of real numbers.

2. If A x B = (p, x); (p, y); (q, x); (q, y)}, find A and B.
Solution:

A is a set of all first entries in ordered pairs in A x B.

B is a set of all second entries in ordered pairs in A x B.

Thus A = {p, q} and B = {x, y}

EXERCISES:
1.Let A = {Elisha, Anna, Mary} and B = {Jonah, Esther}.
a. Find A x B
b. Find B A
2. Let A = {1,2}, B = {x, y}, and C =

c. A x (BnC)

e. (A x B) U (BXC)

9.3. THE GRAPH OF A FUNCTION


Rene Descartes (1596-1650) was a French philosopher and mathematician who is well
known for the famous phrase "cogito ergo sum" (l think, therefore I am), which appears in his
Discours de la methode pour bien conduire sa raison, et chercher la verite dans les sciences
(Discourse on the Method of Rightly Conducting the Reason, and Seeking Truth in the
Sciences). In that same treatise, Descartes introduces his coordinate system, a method for
representing points in the plane via pairs of real numbers. Indeed, the Cartesian plane of modern
day is so named in honor of Rene Descartes, who some call the "Father of Modern
Mathematics."

Descartes' work, which forever linked geometry and algebra, was continued in an appendix to
Discourse on Method, entitled La Geometrie, which some consider the beginning of modern
mathematics. Certainly both Newton and Leibniz, in developing the Calculus, built upon the
foundation provided in this work by Descartes.

84
A Cartesian Coordinate System consists of a pair of axes, usually drawn at right angles to one
another in the plane, one horizontal (labeled x) and one vertical (labeled y), as shown in the
Figure 22.1. The quadrants are numbered l, Il, Ill, and IV, in counterclockwise order, and
samples of ordered pairs of the form (x, y) are shown in each quadrant of the Cartesian
coordinate system in Figure 1 .

11 1

111

Numbering the quadrants. To the right and up is positive,


left and down is negative. FIGURE 1. THE
CARTESIAN COORDINATE SYSTEM.

Now, suppose that we have a relation

Recall that relation is the name given to a collection of ordered pairs. In Figure 2.(b) we've plotted
each of the ordered pairs in the relation R. This is called the graph of the relation R.

9.3.1. Definition:
The graph of relation is the collection of all ordered pairs of the relation. These are usually
represented as points in a Cartesian coordinate system.
y
5

85
11
32

43

5a; (b)

FIGURE 2 A MAPPING DIAGRAM AND ITS GRAPH.


In Figure 2 (a), we've created a mapping diagram of the ordered pairs. Note that the domain
object 3 is paired with two range elements, namely 1 and 4. Hence the relation R is not a
function. It is interesting to note that there are two points in the graph of R in Figure 2. 2(b)
that have the same first coordinate, namely (3, 1) and (3, 4). This is a signal that the graph of
the relation R is not a function. In the next section we will discuss the Vertical Line Test, which
will use this dual use of the first coordinate to determine when a relation is a not a function.

9.32. Creating the Graph of a Function

Some texts will speak of the graph of an equation, such as "Draw the graph of the equation
This instruction raises a number of difficulties.

First, the instruction provides no direction to the reader; that is, what does the instruction
mean? It's not very helpful.
Secondly, the instruction is incorrect. You don't draw the graphs of equations. Rather, you
draw the graphs of relations and/or functions. A graph is just another way of representing
a function, a relation that pairs each element in its domain with exactly one element in its
range.
So, what is the proper instruction? First, we will provide the formal definition of the graph of a
function, then we will break it down by means of examples.

9.3B. Definition:
The graph of a function is the collection of all ordered pairs of the function. These are usually
represented as points in a Cartesian coordinate system.

As an example, consider the function

86
Readers will note that each object in the domain is paired with one and only one object in the
range, as seen in the mapping diagram of Figure 3(a).

Thus, we have two representations of the function f, the collection of ordered pairs (3), and
the mapping diagram of in Figure 3(a). A third representation of the function f is the graph of
the ordered pairs of the function, shown in the Cartesian plane in Figure 3(b).

11
22

33 44
5 (b)

FIGURE 3. A MAPPING DIAGRAM AND ITS GRAPH.

When the function is represented by an equation or formula, then we adjust our definition of its
graph somewhat.

Definition
The graph of f is the set of all ordered pairs (x,f(x))(x,f(x)) so that x is in the domain of f. In symbols,

Graph of f={(x,f(x)):x is in the domain of f.}


This last definition is most easily explained by example. So, let's define a function f that maps
any real number x to the real number x 2; that is, let Now, according to Definition, the graph of f
is the set of all points (x,f(x)), such that x is in the domain of f.

The way is now clear. We begin by creating a table of points (x,f(x)), where x is in the domain
of the function f defined by The choice of x is both subjective and experimental, so we begin by
choosing integer values of x between -3 and 3. We then evaluate the function at each of these x-

87
values (e.g., The results are shown in the table in Figure 4 (a). We then plot the
points in our table in the Cartesian plane as shown in Figure 4(b).

(m,
f(rr))
-3 9
—2 4
—1 1
o
o
1 1
2 4
3
9
(b)

FIGURE 4. PLOTTING PAIRS SATISFYING THE FUNCTIONAL RELATIONSHIP


DEFINED BY THE
EQUATION

Although this is a good start, the graph in Figure 2.2.42.2.4(b) is far from complete. Definition
requires that we plot the ordered pairs (x,f(x)) for every value of x that is in the domain of f.
We've only plotted seven such points, so we're not done. Let's add more points to the graph of f.
We'll evaluate the function at each of the x-values shown in the table in Figure 5(a), then plot the
additional pairs (x,f(x)) from the table in the Cartesian plane, as shown in Figure 50.

a:
—5/2 25/4 (—5/2, 25/4)
—3/2 9/4 (—3/2, 9/4)
(—1/2, 1/4)
—1/2 1/4
(1/2, 1/4)
1/2 1/4 (3/2, 9/4)
3/2 9/4
(5/2,
5/2 25/4
25/4)

(a) (b)
FIGURE 5. PLOTTING ADDITIONAL PAIRS (X, FPO) DEFINED BY THE
EQUATION
We're still not finished, because we've only plotted 13 pairs (x,f(x)), such that Definition 4
requires that we plot the ordered pairs (x,f(x)) for every value of x in the domain off.

However, a pattern is certainly establishing itself, as seen in Figure 5(b). At some point, we
need to "make a leap of faith," and plot all ordered pairs (x,f(x)), such that x is in the domain
off. This is done in Figure 6

88
FIGURE 6. PLOTTING ALL PAIRS (X, F(X)) SO THAT X IS IN THE DOMAIN OF F.
There are several important points we need to make about the final result in Figure 6.

When we draw a smooth curve, such as that shown in Figure 6, it is important to


understand that this is a simply a shortcut for plotting all pairs (x, f(x)), where and x is in
the domain off.
It is important to understand that we are NOT "connecting the dots," neither with a
ruler nor with curved segments. Rather, the curve in Figure 2.2.62.2.6 is the result of
plotting all of the individual pairs (x, f(x)).
The "arrows" at each end of the curve have an important meaning. Much as the ellipsis
at the end of the progression 2,4,6,... mean "et-cetera," the arrows at each end of the
curve have a similar meaning. The arrow at the end of the left-half of the curve
indicates that the graph continues opening upward and to the left, while the arrow at
the end of the right-half of the curve indicates that the graph continues opening
upward and to the right.
Summary:

If a function is defined by an equation, you can create the graph of the function as follows.

1. Select several values of x in the domain of the function f.


2. Use the selected values of x to create a table of pairs (x, f(x)) that satisfy the equation
that defines the function f.
3. Create a Cartesian coordinate system on a sheet of graph paper. Label and scale each
axis, then plot the pairs (x, f(x)) from your table on your coordinate system.
4. If the plotted pairs (x, f(x)) provide enough of a pattern for you to intuit the shape of
the graph of f, make the "leap of faith" and plot all pairs that satisfy the equation

89
defining f by drawing a smooth curve on your coordinate system. Of course, this curve
should contain all previously plotted pairs.
5. If your plotted pairs do not provide enough of a pattern to determine the final shape
of the graph of f, then add more pairs to your table and plot them on your Cartesian
coordinate system. Continue in this manner until you are confident in the shape of
the graph of f.

EXERCISE:
1. Sketch the graph of the function defined by the equation (Given: -2,
-1, 0, 1, and 2
2. Sketch the graph of V'Ñ (Given: 0,1,4,9).
3. Use set-builder and interval notation to describe the domain and range of
the function represented by the graph in Figure below.
y

(a) (b)

4. Use set-builder and interval notation to describe the domain and range of
the function defined by the rule

CHAPTER TEST

a. What can you say about the ordered pairs (a, b) and (b, a)?

b. If A = {p, q, r} and B = {a, b}, find A x B and B x A. Are the two products equal?

c. - { 1, 3, 5} and B = {2, 3}, then Find:


a)

d)

90
d. If A and B are two sets, and A x B consists of 6 elements: If three elements of A x B
are (2, 5) (3, 7) (4, 7) find A x B.

References:

https://www.math-only-math.com/ordered-pair.html

https://www.toppr.com/guides/+maths/relations-and-functions/cartesian-product-sets/

https://www.math-only-math.com/Cartesian-Product-of-Two-Sets.html

https://www.math-only-math.com/worksheet-on-math-relation.html
https://www.splashlearn.com/math-vocabulary/geometry/ordered-pair
https://math.libretexts.org/Bookshelves/Algebra/Book%3A Intermediate Algebra (Arnold)/02
0
/63A Functions/2.02%3A The Graph of a Function

https://math.libretexts.org/Bookshelves/Algebra/Book%3A Intermediate Algebra (Arnold)/02


0
/63A Functions/2.03%3A Interpreting the Graph of a Function

CHAPTER 10
Refatíons
Introduction:
According to Hume, the mind is capable of apprehending two kinds of proposition or
truth: those expressing "relations of ideas" and those expressing "matters of fact." The former
can be intuited—i.e., seen directly—or deduced from other propositions. This is a relation of
ideas and matter of fact.

In mathematics, a relation is an association between, or property of, various objects.


Relations can be represented by sets of ordered pairs (a, b) where a bears a relation to b. Sets
of ordered pairs are commonly used to represent relations depicted on charts and graphs, on
which, for example, calendar years may be paired with automobile production figures, weeks
with stock market averages, and days with average temperatures.
This chapter presents the concepts of relation, the properties of relation, the relation
presented by digraph, the equivalence and partial order relation, and the Well Ordering
Principle says that N is well ordered under the "g" relation.

91
Lecture NoteS
10.1 Relation
A relation on a set A means a subset of A x A. For example, if A = {1, 2, 3} then the
following are some, but not all, possible relations on A. 1) R = {(1,1), (12), (1,3)}
2) R = {(2,3)}
3) R = {(1,1), (1,2), (1,3), (22), (2,3), (3,3)}

• If R is a relation on A then the inverse of R is the relation R


Furthermore, if S is another relation on A then the composition of R with S is the relation S 0
R = {(a, c) I (a, b) ER (A b, c) eS}. In particular, we define R2 = R 0 R, R3 = R2 0 R, etc.

Example:

Let A = {1, 2, 3, 4} and R = {(1,2), (2,3), (2,4), (3,3), (41)} and S = {(1,3), (22), (3,1), (3,3)}. Then

• R-1 = {(2,1), (3,2), (3,3), (IA)}

• S o R = {(1,2), (2/1), (2B), (3, l), (3/3), (43)}

• R o S = {(1,3), (2B), (2,4), (32), (3,3)}

• R2 = R O R = {(1,3), (2,3), (2,1), (3,3), (42)}

EXERCISE:
If IAI = n then how many different relations on A are possible?
2. Let A = {1, 2, 3, 4} and R = {(1,2), (2,1), (2,4), (3,3), (41), (43)} CA x A.
a) Find RI and (R -l) -l
b) Find R 2 and R 3
3. Prove that R 0 (R 0 R) = (R 0 R) 0 R. Hence we may write R 3 = R O R 0 R.

10.2 Properties of a relation R GA x A.


1) reflexive if V A (a, a)

2) symmetric if V a GA, (a, b) (b, a)

92
3) anti—symmetric if V a GA, (a, b) ( A b, a) ab
4) transitive if Va, b, c GA, (a, b) ( A b, c) (a, c)
Example:

Let A = {1, 2, 3} and consider three relations on A:


R = {(1,1), (1,2), (2,1), (2,2), (3/3)}
S = {(1,1), (1B), (22), (3,2)}

T = {(1,2), (1,3), (2,3)}


R is reflexive, symmetric, and transitive, but not anti-symmetric.
S is anti-symmetric, but not reflexive, not symmetric, and not transitive. T is anti-
symmetric and transitive, but not reflexive and not symmetric.

EXERCISE:
4. Let A = {1, 2, 3, 4}. Which properties above are true for each relation R on A?

b) R = {(1,1), (1,2), (1,3), (1,4), (2,2), (2/4), (3,3), (44)}


5. Let A = {1, 2, 3, 4}. Give any example of a relation R on A which is
a) reflexive, not anti-symmetric, not transitive.
b) not reflexive, not symmetric, not transitive.
c) symmetric and transitive.
6. Let R be a relation on A. Prove the following propositions.
a) R is symmetric if and only if R -l = R

10.3 RELATION REPRESENTED BY DIAGRAPH


A relation R C A x A can be represented by a digraph in which each element of A is
represented by a vertex and each element (a, b) e R is represented by an edge with direction
from a to b. In the case a = b the edge is called a loop.

E
xample: A = {l, 2, 3, 4} and R

93
EXERCISE
8. How can you tell from the digraph if R is
a) reflexive

b) anti-reflexive [meaning that a V EA (a, a) eR]

10.4. Rç A x A Is Called An Equivalence Relation If It Is Reflexive, Symmetric, And Transitive.


10.41 If R is an equivalence relation on A then A is partitioned into subsets or classes of the
forms Ax = {a EA I (a, x) eR} for every x EA. These subsets of A are called the
equivalence classes of A under R and they satisfy the following properties.

1) (x, y) AX Ay
2) (x, y) AX n Ay (P

Example: The following digraph shows that R is an equivalence relation. (Why?) There are

three equivalence classes namely

RCA x A is called a B if it is reflexive, anti-symmetric, and transitive.

10.42. If R is a partial order relation then its digraph can be simplified into a Hasse diagram
after these four steps:

1) Do not draw loops.


2) Do not draw (a, c) whenever there are (a, b) and (b, c). 3) Redraw the
remaining graph so that all edges point upward.
4) Do not draw the directions.
Example: The following digraph shows that R is a partial order relation. (Why?) The four steps
above lead to the Hasse diagram of R.

94
10.43. A partial order relation R on A is called a total ordering if it satisfies one additional
proposition: Va EA Vb GA, (a, b) ER V (b, a) CR.
Suppose R is a partial order relation on the set A. An element I EA is called a least element
under R if Va GA, (l, a) GR. Now R is called a well ordering on A if every non-empty subset of A
has a least element.

EXERCISE
9. Prove that the following relations are equivalence relations.

b) R = {(a, b) EA x A I a mod 3 = b mod 3} where A = {0, 1, 2, 9}


10. Find the equivalence classes for each relation in Problem 9.
12. Prove that the following relations are partial ordering.

a) A = {50, 22, 35, 14} and R (a, b) x A l a b}


b) A = {1, 2, 6, 12, 24} and R = {(a, b) EA x A l a divides b}
15. Prove that the relation a b gives a total ordering on R.

10.5. The Well Ordering Principle says that N is well ordered under the relation.
Theorem: The Well Ordering Principle is equivalent to the Principle of Mathematical
Induction.
n} then a relation R C A x A can be represented by a zero-one matrix M of size
n x n where (M)ij = 1 if (i,j) GR and (M)ij = 0 if (i, j) eR.
Example. Suppose A = {1, 2, 3} and R = {(1,1), (1,3), (2/1), (32), (3,3)}. Then the
1 0 1 zero-one matrix of R is M = 1 0 0

011
EXERCISE
20. Convert these zero-one matrices to digraphs.

95
1 o 1O oo o1 o 1 o 1 O1 o o

10.6. Transitive Closure


The transitive closure of R CA x A is the smallest transitive relation containing R.
Theorem: The transitive closure of R is given by R UR2 u... u Rn where n = IAI

EXERCISE

21. Let A = {1, 2, 3, 4}. Use this theorem to find the transitive closure of R CA x A.

22. Find the zero-one matrix of the transitive closure for each R of the following:

1 o 10 0 0 o 1 b) 1 0 1 o 1 O1 0 0

CHAPTER TEST:
1.Let A = {1, 2, 3, 4} and R = {(1,2), (2, 1), (2,4), (3,3), (41), (4/3)} CA x A.

a. Find R 0 R -l and R I O R

b. Find (R -1)2 and (R2) -l

2. Let A = {1, 2, 3, 4}. Which properties above are true for each relation R on A?
a. R = {(1,1), (1/3), (2,1), (22), (2,4)}

3. Let A = {1, 2, 3, 4}. Give any example of a relation R on A which is


a. neither symmetric nor anti-symmetric.
b. both symmetric and anti-symmetric.

96
4. Let R be a relation on A. Prove the following propositions.
a) R is anti-symmetric if and only if R n R -l {(C a, a) I a GA}.
b) R is transitive if and only if R 2 CR
5. Draw the digraph for each of the relations in Problem 2.

6. How can you tell from the digraph if R is

a. symmetric
b. anti-symmetric
c. transitive
6. Prove that the following relations are equivalence relations

a) R = {(a, b) e Z x Z I a mod 5 = b mod 5}


b) R = b) EA x A l a = b} where A = {1, 2, 3, 4}
c) R = {(a, b) e Z x Z l a + b is even}
7. Find the equivalence classes for each relation in Problem 6.
8. Define the congruence relation on Z by a b if and only if a mod n = b mod n. Let R = {(a, b) e
Z x Z I a b}. Prove that R is an equivalence relation on Z and find the equivalence classes.

9. Prove that the following relations are partial ordering.


a) A = {2, 3, 6, 10, 20, 30} and R = {(a, b) EA x A I a divides b}
b) R = {(a, b) e N x N I a divides b}
10. Draw the Hasse diagram for each partial order relation in Problem 9.

11. Prove that a well ordering is a total ordering but not conversely.

12. Give an example of a total ordering on a set which is not a well ordering.
13. Represent the relations given in Problem 2 using zero-one matrices.

14. Let A = {1, 2, 3, 4}. Use this theorem to find the transitive closure of R CA x A.

References:
https://www.britannica.com/topic/epistemology/David-Hume#ref309000
https://www.britannica.com/science/set-theory/Operations-on-sets#ref387950

97

You might also like